Sei sulla pagina 1di 73

Parliament of India

Indian democracy is based on the Westminster model (British model of democracy is referred to as the Westminster model) where the importance of Parliament in the political system is central. Preamble to the Indian Constitution begins with the people. which confers sovereignty on the Parliament as people in an indirect democracy means the representative body. Art. 79 says that there shall be a Parliament for the Union which shall consist of the President and two Houses to be known as the Rajya Sabha or the federal chamber or Council of States or Upper House and the Lok Sabha or the popular chamber or Lower House or House of the 'People'. Even though the President of India is not a member of the parliament, he is a part of the Parliament for the following reasons

in a parliamentary system, the Executive is a part of the Legislature unlike the Presidential form of democracy where there is a strict separation between the two institutions. Bills passed by the Parliament need Presidential assent before they become laws President performs certain other legislative duties like summoning and proroguing the Parliament; recommending the introduction of certain Bills in the Parliament etc.

The term in Constitutional law, President-in-Parliament, is used to refer to the President in his legislative role, acting with the advice and consent of the two Houses of Parliament. It is similar to Crown-in-Parliament which means the Crown acting with the advice and consent of the British Parliament. Role of the President of India with regard to Parliament The President is the constitutional head of Republic of India. He is elected by an electoral college that includes elected members of both-Houses of Parliament and the elected members of the Legislative Assemblies of the States. The President performs the following constitutional functions vis-a-vis Parliament

He invites the leader of the majority party to form the Government after a new Lok Sabha is duly elected. He summons the two Houses of Parliament to meet from time to time. He has the power to prorogue a session in the two Houses and dissolve the Lok Sabha. The President has to assent to a Bill before it can become a law. If the Houses are not in session, the President can promulgate Ordinances having the same validity as a law passed in Parliament. The President has the right to address either or both houses of Parliament. The President has the power to call both houses for a joint Sitting in case a dispute arises over passing a Bill.

He nominates 12 members of the Rajya Sabha and has the right to nominate two members from the Anglo Indian community to the Lok Sabha if they are under represented.

Thus, President is a constituent part o the Parliament. Ques. 1 : Rajya Sabha has an insignificant role in the functioningof parliamentary democracy in India. Critically examine? Ans. It is the federal house representing the States. Maximum strength (sanctioned strength) of Rajya Sabha is two hundred and fifty (250), of which 238 are to be elected and 12 are nominated by the President of India. The actual strength of Rajya Sabha is two hundred and forty five (245), of which 233 are elected and 12 are nominated by the President. The actual strength is also known as total membership. Each State and the two Union Territories of Delhi and Puduchery are represented in the Rajya Sabha. The allocation of seats in Rajya Sabha is contained in the Fourth Schedule to the Constitution. Article 80 provides that the Rajya Sabha shall consist of:

twelve members nominated by the President from amongst persons having special knowledge or practical experience in respect of such matters as Literature, science, art and social service; and not more than two hundred and thirty-eight representatives of the States and of the Union Territories.

The elected members of the (233 Members) Rajya Sabha are elected by the elected members of the Assemblies of States and the two Union Territories of Delhi and Puduchery in accordance with the system of proportional representation by means of the single transferable vote. Of the UTs, only NCT of Delhi and Puduchery are represented in the Rajya Sabba. No other UT has an assembly and so has any representation in the Rajya Sabha. While the nominated members of Rajya Sabha have a right to vote in the election of the VicePresident of India, they are not entitled to vote in the election of the President of India. The Council of States was set up under the Constitution in 1952. Dr. Radhakrishnan was the first Chairman of Rajya Sabha. He was the longest serving Chairman (1952-62). The allocation of seats to be filled by the representatives of the States/Union Territories as laid down in the Fourth Schedule to the Constitution is as follows: 1. 2. 3. 4. 5. Andhra Pradesh Arunachal Pradesh Assam Bihar Chhattisgarh 18 1 7 16 5

6. 7. 8. 9. 10. 11. 12. 13. 14. 15. 16. 17. 18. 19. 20. 21. 22. 23. 24. 25.

Goa Gujarat Haryana Himachal Pradesh Jammu and Kashmir Jharkhand Karnataka Kerala Madhya Pradesh Maharashtra Manipur Meghalaya Mizoram Nagaland Orissa Punjab Rajasthan Sikkim Tarnil Nadu Tripura

1 11 5 3 4 6 12 9 11 19 1 1 1 1 10 7 10 1 18 1

26. 27. 28. 29. 30. Total:

Uttaranchal Uttar Pradesh West Bengal

3 31 16

The National Capital Territory of Delhi 3 Puduchery 1 233

Eligibility A candidate for election to Rajya Sabha


should be a citizen of India above 30 years of age and possessing such other qualifications as may be prescribed by law of Parliament

Rajya Sabha is not subject to dissolution; one-third of its members retire every two years. Thus, it is a permanent body. Normally a member is elected for six years but a member elected against a mid-term vacancy (casual vacancy), serves only for the remaining period. Rajya Sabha, when it was constituted in 1952, had 2l6 members-12 nominated by the President and the remaining 204 elected to represent States. President, after consultation with the Election Commission made an order in 1952 for curtailing the term of office of some of the members so that as nearly as one-third of the members retire after every two years. Election Commission by drawing of lots decided who should retire and when. That is how the initial order was established. Ques. 2 : The amendments in the Representation of Peoples Act, 1957 related to Rajya Sabha is violates the basic structure of the constitution as federalism is a basic feature. Examine Ans. In 2003, Parliament made an amendment to the Representation of Peoples Act, 1951 to make two crucial changes to do away with the domicile / residency condition to replace secret ballot with open ballot. Section 3 of the Representation of the People Act said that a candidate seeking election to Rajya Sabha should be ordinarily resident in the State that he wants to represent. By amending this Section of the RPA, the Government opens the contest for a resident anywhere in the country. Government explained that the purpose of the first change was to remove the anomaly in the eligibility criteria for both the Houses of Parliament a candidate for Lok Sabha can contest from anywhere in the country but it is not so for Rajya Sabha.

The residency rule for the Rajya Sabha became controversial when the Election Commission questioned the genuineness of the domiciliary credentials of some members of Rajya Sabha. In a large number of cases, representatives from various Sates in Rajya Sabha were those who traditionally were not resident in that State, but for the purposes of election to the Rajya Sabha, got enrolled as voters in that particular State by acquiring property or otherwise. The residency clause was flouted frequently by many. Regarding adoption of open ballot, the reason is: in the context of the growing money power in RS elections, secrecy was thought to conceal corruption and so open ballot was introduced. The amendments were challenged in the Supreme Court on the ground that basic Structure of the Constitution is violated as federalism is a basic feature. It is argued that only those belonging to a State can represent it well. In 2006 a five-judge Constitution bench of the apex court in the Kuldip Nayyar Vs Union of India (2006) case gave the following verdict

residence is not a constitutional requirement but a matter of qualification prescribed by Parliament in exercise of its power under Article 84 and so the question of violation of basic structure does not arise as long as the state has a right to be represented in the council of states by its chosen representative, who is a citizen of the country, it cannot be said that federalism is affected. Constitution does not provide that voting for an election to the Council of States shall be by secret ballot. The manner of voting in the election to the Council of States can be regulated by the statute.

The court said that since the amendments have been brought in to avoid cross- voting, to wipe out the evil of corruption and to maintain the integrity of the democratic set-up, they can be justified by the State as a reasonable restriction under Article-19(2) of the Constitution on the assumption that voting in such an. election amounts to freedom of expression under Article 19(l)-A. Ques. 3 : The role of importance of the Chairman of the Rajya Sabha? Ans. The Vice-President is the ex officio Chairman of Rajya Sabha (Art.64). In fact, the Vice President draws his salary as the Chairman of the Rajya Sabha which is his ex-officio role that is, by virtue of being the Vice president of India, he functions as the Chairman of Rajya Sabha. The Vice-President is elected by the members of an electoral college consisting of all the members of both Houses of Parliament both elected and nominated in accordance with the system of proportional representation by means of the single transferable vote. The Vice-President holds office for a term of five years from the date on which he enters upon his office. As the Presiding Officer, the Chairman of the Rajya Sabha is the guardian of the prestige and dignity of the House. He safeguards the privileges and immunities of the members individually and the House collectively. He issues warrants to execute the orders of the House, where necessary. For example, to punish any one who commits contempt of House.

Under the Constitution, the Chairman exercises only a casting vote in the case of equality of votes (Art 100.1) However, during proceedings for his removal; he does not preside at that sifting. He cannot also vote at all on such resolution. The Constitution also lays down certain powers and duties of the Chairman

he is empowered to adjourn the House or to suspend its sitting in the event of absence of quorum In case of resignation of a member from the House, the Chairman is required not to accept the resignation, if he is satisfied that such resignation is not voluntary or genuine under the Tenth Schedule to the Constitution, the Chairman determines the question as to disqualification of a member of the Rajya Sabha on ground of defection. He also makes rules for giving effect to the provisions of that Schedule enforce respect for privileges and the Chairman may permit a member who is unable to express himself in Hindi or in English, to address the House in his mother tongue

Various powers are conferred on the Chairman under the Rules of Procedure of the Rajya Sabha in connection admissibility of motions etc. The Chairmans consent is required to raise a question of breach of privilege in the House. Parliamentary Committees where members are drawn from Rajya Sabha, whether set up by the Chairman or by the House, work under his guidance. He appoints Chairmen and nominates members to eight Departmentally related Standing Committees and they are under his administrative control. He himself is the Chairman of the Business Advisory Committee, Rules Committee and the General Purposes Committee. The Chairmans rulings cannot be questioned or criticised and to protest against - the ruling of the Chairman is a contempt of the House and the Chairman. The Chairman does not take part in the deliberations of the House except in the discharge of his duties as the Presiding Officer. However, on a point of order raised or on his own, he may address the House at any time on a matter under consideration with a view to assisting members in their deliberations. Maintenance of order in the House is a fundamental duty of the Chairman and he has disciplinary powers like suspension of member; and may also adjourn the sitting of the house in case of grave disorder. Some statutes also confer duties on the Chairman

rules made under the Salary, Allowances and Pension of Members of Parliament Act, 1954, do not take effect until they are approved and confirmed by the Chairman and the Speaker Under the Judges (inquiry) Act, 1968, the Chairman has to constitute a Committee, upon receipt of a motion for the removal of a Judge of the Supreme Court or of a High Court, for investigation into the grounds on which the removal of a Judge is prayed for.

The Rajya Sabha Secretariat functions under the control and direction of the Chairman.

Deputy Chairman The Deputy Chairman is elected by the members of Rajya Sabha from among themselves. While the office of Chairman is vacant, or during any period when the Vice-President is acting as, or discharging the functions of the President, the duties of the office of the Chairman are performed by the Deputy Chairman. He/She has - the same powers as the Chairman when presiding over a sitting of the House. The Deputy Chairman can speak in the House, take part in its deliberations and vote as a member on any question before the House, but he/she can do so only when the Chairman is presiding. When the Deputy Chairman himself/herself is presiding, he/she cannot vote except in the event of equality of votes-casting vote to break the tie. The Deputy Chairman holds office from the date of his/her election and vacates the office if he/she ceases to be a member of the House. He/She may resign his/her addressing the letter to the Chairman. The Deputy Chairman may also be removed from his/her office by a resolution of the House passed by a majority of all the then members of the House. Fourteen days notice is required of the intention to move such a resolution. The salary of the Deputy Chairman is charged on the Consolidated Fund of India and is not subject to the vote of the House. Chairman Protem When the offices of both the Chairman and the Deputy Chairman are vacant, the duties of the office of the Chairman arc performed by such member of the Rajya Sabha as the President may appoint for the purpose (Art.91) The member so appointed is known as the Chairman protem. For the first time in the Rajya Sabha when the Vice-President (Shri B. D. Jatti) was acting as the President and the post of Deputy Chairman held by Shri Godey Murahari having fallen vacant in 1977 as the latter was elected to the Lok Sabha, the Vice-President acting as President appointed Shri Banarsi Das, a member of Rajya Sabha, as Chairman protem until the Deputy Chairman was chosen the election of the Deputy Chairman took place soon after and the protem Chairman vacated the office. Leader of the Opposition is the leader in the Raya Sabha of the party in opposition to the Government having the greatest numerical strength and recognised as such by the Chairman of the Rajya Sabha. Panel of Vice-Chairmen The Chairman, from time to time, nominates from amongst the members of the house, a panel of not more than six Vice-Chairmen. In the absence of the Chairman and the Deputy Chairman, one of them presides over the House. The Vice-Chairman, when presiding over a sitting of the House, has the same powers as the Chairman when so presiding. He is, however, free to participate fully in all discussions in the House. A Vice-Chairman while presiding cannot vote in the first instance, and has to exercise a casting vote in the case of an equality of votes. Non-panel Member Presiding When neither the Chairman nor the Deputy Chairman nor a Vice-Chairman is present to preside, such other member as may be determined by the House acts as the Chairman. The practice is that

the outgoing presiding officer requests a member to take the Chair with the approval of the House. Leader of the House The Leader of the House is an important parliamentary functionary who assists the Presiding Officer in the conduct of the business. Leader of Rajya Sabha is the Prime Minister, if he is a member of the House, or a Minister who is the member of the House and nominated by the Prime Minister to function as the Leader of the house. The Prime Minister, Manmohan Singh, who is a Rajya Sabha member, is the leader of Rajya Sabha while the former Union Minister, Arun Jaitley is the leader of Opposition. Casting vote Casting vote is the vote cast to break a tie when there is equality of votes. Under the Constitution, the Chairman exercises only a casting vote in the case of equality of votes. However, if at any sitting of the House a resolution for the removal of the Chairman from his office is under consideration, he is not to preside at that sitting. He cannot also vote at all on such resolution or on any other matter during such proceedings. While the former (casting vote being given) helps in the conduct of business and Legislation, the latter (casting vote being denied) is a moral imperative. Utility of Rajya Sabha

It is the permanent House and so has benefits like it can ratify proclamation of Emergency when the Lok Sabha is not in session and cannot be called into session immediately. It means the proclamation can continue. At the same time, it can ensure that emergency provisions are not misused- particularly the Presidents rule. Unless Rajya Sabha ratifies the proclamation of Presidents rule, it does not come into force. Thus, the interests of the States can be protected. Constitution can not be amended unless Rajya Sabha, sitting independently of the Lok Sabha passes the Bill. That is, there is no joint session in case of deadlock. Thus, the national and states interests are protected. It has 12 nominated members who add to the quality of parliamentary proceedings and policy making It enables law making to become more sober when the representatives of the people are carried away by emotional issues.

Thus, Rajya Sabha has enormous utility, in. the Parliamentary democracy of ours. Non-federal features of Rajya Sabha The non-federal features of the Rajya Sabba are All states do not have same number of representatives in the Rajya Sabha like in the US Senate. Rajya Sabha has no special powers with regard to creation of states (Art.3) and thus can not defend the principle of indestructibility of the state concerned. Art.249 is also unfederal as the big 10 states can over run the rest while passing the resolution thus denying equality to states.

Vote in the House In Rajya Sabha, as in Lok Sabha generally four methods of voting are adopted

Voice vote Counting Division by going into the Lobbies Division by automatic vote recorder

Voice vote On the conclusion of a debate, the Chairman puts the question before the House and invites those who are in favour of the motion to say Aye and those against the motion to say NO Then the Chairman says: I think the Ayes or the Noes, (as the case may be) have it. Count If the opinion of the Chairman as to the decision of a question is challenged, he may, if he thinks fit, ask the members who are for Aye and those for No respectively to rise in their places and, on a count being taken, he may declare the determination of the House. In this case, also, the names of the voters are not recorded. Division A division is one of the forms in which the decision of the House is ascertained. As mentioned above, normally, when a motion is put to the House members for and against it indicate their opinion by sing Aye or No from their seats. The Chairman goes by the voices and declares that the motion is either accepted or negatived by the House. If a member challenges the decision, the Chair orders that the lobbies be cleared. Then tie division bell is rung. After the bell stops, all the doors-to the Chamber are closed and nobody can enter or leave the Chamber till the division is over. Then the Chairman puts the question for second time and declares whether in its opinion the -Ayes or the Noes, have it. When a division is about to be taken, only members of the House have the right to be present. In other words, the lobby has to be cleared for a division. If the opinion so declared is again challenged, the Chair asks the votes to be recorded by operating the Automatic Vote Recording Equipment. Special Mention Under the Rules of Procedures and Conduct of Business in the Council of States, members are allowed to make special Mentions in Rajya Sabha. If a Minister so desires, he may make a statement on the subject with the permission of the Presiding Officer. The main advantage of this device is to bring to the notice of the House and the Government the matters and happenings of urgent public importance which take place in or outside the country. Motion for Papers There is no provision for adjournment motion in Rajya Sabha as the Council of Ministers is responsible only to the Lok Sabha (Art. 75.3). But there is a motion for papers, like in the

House of Lords in Britain. Adjournment motion sets aside the announced business of the House in preference for discussion on a matter of urgent public importance which in the end is voted upon. Under a motion for papers, the Council could discuss any matter of real public importance and the right of the reply is given to the member moving the motion.. LOK SABHA Lok Sabha is composed of representatives of the people chosen by direct election on the basis of universal adult suffrage. The maximum strength of the House envisaged by the Constitution is 552-upto 530 members to represent the States, up to 20 members to represent the Union Territories and not more than two members of the Anglo-Indian Community to be nominated by the President, if, in his opinion, the community is not adequately represented in the House. The total elective membership is distributed among the States in such a way that the ratio between the number of seats allotted to each State and the population of the State is, so far as practicable, the same for all States. The actual strength of the Lok Sabha at present is 545 members including the Speaker and two nominated members referred to as the total membership. Lok Sabha, unless sooner dissolved, continues for five years from the date appointed for its first meeting. However, while a Proclamation of Emergency is in operation, this -period may be extended by Parliament for a period not exceeding one year at a time and not extending beyond a period of six months after the proclamation has ceased to operate. The Constitution of India came into force on January 26, 1950. The first general elections under the new Constitution were held during the year 1951-52 and the first elected Parliament came into being in April, 1 952, the Second Lok Sabha in April,1957, the Third Lok Sabha in April,1962, the Fourth Lok Sabha in March, 1967, the Fifth Lok Sabha in March, 1971, the Sixth Lok Sabha in March, 1977, the Seventh Lok Sabha in January, 1980, the Eighth Lok Sabha in December, 1984, the Ninth Lok Sabha in December, 1989, and the Tenth Lok Sabha in June, 1991, the Eleventh Lok Sabha in May, 1996, the Twelfth Lok Sabha in March, 1998 ,the 13th Lok Sabha in late 1999 and the 14th Lok Sabha elections were held in 2004. Presiding Officers Lok Sabha elects one of its own members as its Presiding Officer and he is called the Speaker. He is assisted by the Deputy Speaker who is also elected by Lok Sabha. The conduct of business in Lok Sabha is the responsibility of the Speaker. Ques. 4 : The Speaker of Lok Sabha has extensive functions to perform in matters of administrative, judicial and regulatory. Discuss? Ans. In the Lok Sabha, both Presiding Officers-the Speaker and the Deputy Speaker- are elected from among its members by a simple majority of members present and voting in the House. No specific qualifications are prescribed for being elected the Speaker. The Constitution only requires that he should be a member of the House. One of the first acts of a newly constituted House is to elect the Speaker. Usually, a member belonging to the ruling party is elected the Speaker. But in times of coalition governments, as in India since 1996, a member of a party other than the ruling coalition can be elected the Speaker. For example, Somnath Chatterjee of CPI(M), who belong to a party that only gives outside support to the coalition.

The Speaker protem (sworn in by the President to swear in the newly elected members of House) presides over the sitting in which the Speaker is elected, if it is a newly constituted House. If the election falls later in the life of a Lok Sabha the Deputy Speaker presides. Term of Office The Speaker holds office from the date of his election till, immediately before the first meeting of the Lok Sabha which is newly constituted after the dissolution of the one to which he was elected. He is eligible for re-election. On the dissolution of the Lok Sabha, although the Speaker ceases to be a member of the House, he does not vacate his office. The Speaker may, at any time resign from office by writing under his hand to the Deputy Speaker. The Speaker can be removed from office only on a resolution of the House passed by a majority of all the then members of the House. It is mandatory to give a minimum of 14 days notice of the intention to move the resolution. At the Commencement of the House or twin time to time, the Speaker shall nominate from amongst the members a panel of not more than ten Chairmen, anyone of whom may preside over the House in the absence of the Speaker and the Deputy Speaker. The Speaker has extensive functions to perform in matters administrative, judicial and regulatory. His decisions are final and binding. Under the Constitution, the Speaker enjoys a special position

he certifies Money Bills and is final (Art. 110) presides over joint sittings which are summoned to resolve a disagreement between the two Houses decides on granting recognition to the Leader of the Opposition in the Lok Sabha Following the 52nd Constitution amendment 1985, the Speaker is vested with the power relating to the disqualification of a member of the Lok Sabha on grounds of defection.

Though himself a member of the House, the Speaker does not vote in the House those rare occasions when there is a tie- equality of votes. Till date, the Speaker of the Lok Sabha has not been called upon to exercise this unique casting vote. Speaker and the Committees The Committees, constituted by him or by the House function under the overall direction of the Speaker. The Chairmen of most Parliamentary Committees are nominated by him. Committees like the Business Advisory Committee, the General Purposes Committee and the Rules Committee work directly under his Chairmanship. He is the ex officio President of the Indian Parliamentary Group (IPG), .set up in 1949, which functions as the National Group of the Inter-Parliamentary Union (IPU) and the Main Branch of the Commonwealth Parliamentary Association (CPA). It has been said of the office of the Speaker that while the members of Parliament represent the individual constituencies, the Speaker represents the full authority of the House itself. He symbolises the dignity and power of the House. His unique position is illustrated by the fact that he is placed very high in the Warrant of Precedence in our country, standing next only to the President, the Vice-President and the Prime Minister. Speakers salary and allowances are charged on the Consolidated Fund of India.

Ques. 5 : Briefly write about (a) Protem Speaker (b) Substantive Motions (c) Difference between a resolution and a motion.

Ans. Protem Speaker Speaker pro Tempore or temporary Speaker the senior most member of the Lok Sabha is appointed and sworn in as the Protem Speaker by the President so that the newly elected members are administered oath by him. They in turn elect the Speaker. Protem Speaker continues till the new Speaker is elected. The Constitution does not specify any functions for the Protem Speaker. Somnath Chatterjee who is the senior most MP of the 14th Lok Sabha was administered oath as protem Speaker by President A.P.J. Abdul Kalam in June 2004 , enabling him to preside over the proceedings of the first two days of the House when new members took oath. He was later elected Speaker of the House when he ceased to be the protem Speaker. Procedure in the House The Rules of Procedure and Conduct of Business in Lok Sabha and Directions issued by the Speaker from time to time regulate the procedure in Lok Sabha. For Various items of business to be taken up in the House the time is allotted on the recommendations of the Business Advisory Committee. Time of Sittings When in session, Lok Sabha holds its sittings usually from 11 AM. to 1 P.M. and from 2 P.M. to 6 P.M. Question hour It is the beginning hour of the Parliament on Zero hour Zero hour has no basis in the Parliamentary rules. It developed by convention to enable members to raise matters of public importance on the floor of the House. Zero Hour immediately follows question hour. It begins at 12 oclock after Question Hour which is from 11.00 a.m. to 12.00 noon. Though called although euphemistically called Zero Hour, it may last for more or less than an hour. Zero hour is observed in both the Houses of the Parliament. Motions and Resolutions Motion: Process of passage and Different types of Motions

A motion is a proposal for eliciting decision or expressing the opinion of the house on a matter of public importance. Every question to be decided by the House must be proposed as Motion. The consent of the Presiding Officer is essential to initiate a motion. A motion is so called as it sets the House in motion the business of the House essentially takes place on the basis of motions. Government motions involve seeking approval of the House for a policy of the government Private members motions focus on eliciting opinion of the House on a particular matter. Motions fall into three principal categories:

Substantive Motions Substitute Motions Subsidiary Motions.

A substantive motion is a self-contained independent proposal. It is drafted in such a way as to be capable of expressing a decision of the House. Some examples of a substantive motion are: the motion of thanks on the Presidents Address, motion of no-confidence, motions for elections, motion for impeachment of persons in high authority. A substitute motion is moved in the place of the original motion. It proposes an alternative to the original motion. Resolutions A Resolution is a procedural means to initiate a discussion on any matters of general public interest. A Resolution is actually a Substantive Motion. Resolutions may be classified as private members resolutions, government resolutions and statutory resolutions provision in the Constitution or an Act of Parliament. Government resolutions are initiated by ministers. Statutory resolutions may be moved either by a minister or by a private member. Difference between a motion and a resolution. All Resolutions fall in the category of Substantive Motions. But all motions need not necessarily be substantive. Further, all motions are not necessarily put to vote of the House, whereas all the resolutions are required to be voted upon. For example, the resolution to impeach the President of India. Ques. 6 : Though according to the Constitution both Lok Sabha and Rajya Sabha enjoy equal status but Lok Sabha is relatively more powerful. Examine? Ans. The Constitution envisages that both Lok Sabha and Rajya Sabba have equal status and position. The two houses, at the same time enjoy special powers as given below The Lok Sabha has the following special or exclusive powers Union Council of Ministers is collective responsible to Lok Sabha (Art.75.3)

Budget is presented in Lok Sabha (Art. 112) demands for grants can be introduced only in the Lok Sabha Money Bill (Art.110) or a Financial Bill (Art.117.1) can be introduced only in the Lok Sabha.

Speakers decision about whether a Bill is a Money Bill or not is final Prime Minister generally comes from the Lok Sabha Estimates Committee has its entire 30 members drawn from the Lok Sabha. Lok Sabha has 545 members which is substantially more than that of Rajya Sabha. Its numerical Superiority helps in the joint session of the Parliament which is presided over by the Speaker A joint session is presided over by the Speaker and in his absence the deputy Speaker of Lok Sabha Lok Sabha has the power of moving a resolution for the discontinuation of national emergency as provided by the 44th Amendment Act (Art 352) The Rajya Sabha has special or exclusive powers which are contained in Articles 249, 312, 352, 356 and 360. Under article 249, the Rajya Sabha can enable he Parliament, by passing a resolution supported by two-thirds of the members present and voting, that Parliament should make laws with respect to any matter enumerated in the State List specified in the resolution, in national interest. resolution can be passed by the Rajya Sabha by a majority of 2/3rds of the members present and voting, under Article 312, for the creation of one or more all-India services by the Parliament, if it is deemed to serve the national interest. The services such as the Indian Administrative Service, Indian Police Service, Indian Forest service and All-India Judicial Service are the All India Services. Under articles 352, 356 and 360, the Rajya Sabha can approve the Proclamations of emergency national, state and financial respectively initially or extend them subsequently while the Lok Sabha is not in session or under dissolution. The Constitution requires the laying of a number of papers on the Table in both the Houses, notably amongst them are the Budget, supplementary demands for grants, Ordinances and Proclamations issued by the President, reports of Constitutional and statutory functionaries such as the Comptroller and Auditor-General, the Finance Commission, the Commissioners for the Scheduled Castes and Scheduled Tribes, the Backward Classes Commission, the Commissioner for Linguistic Minorities and the Union Finance Commission Both Houses also participate in matters of elections of the President and the VicePresident Both participate in impeachment of the President, a Judge of the Supreme Court or of a High Court and CAG.

Except the above, there is equality between the two Houses:

Differences between Lok Sabha and Rajya Sabha The following are the differences Members of Lok Sabha are directly elected on the basis of universal adult franchise. Members of

Rajya Sabha are elected by the elected members of State Assemblies in accordance with the system of proportional representation by means of the single transferable vote

The normal life of every Lok Sabha is 5 years while Rajya Sabha is a permanent body Rajya Sabha has a nominated component- 12 members of intelligentsia-which Lok Sabha does not have

Vacation of Seats If a member of one House becomes a member of the other House, his seat in the first House becomes vacant from the date on which he is elected to the other House. If he is elected a member of the state Legislature, he ceases to remain a member of Parliament, unless he resigns his membership from the State Legislature within a period of 14 days from the date of publication of the result in the State Gazette. If a person is chosen a member of both the Houses but has not taken his seat in either of them, then he has to intimate in writing to the Election Commission, within ten days of the publication of the result as to in which House he wishes to serve and thereupon his seat in the other House becomes vacant. If he fails to give such intimation, his seat in the Rajya Sabha becomes vacant after the expiration of that period. If a person is elected to more than one seat in the House, then all the seats become vacant unless he resigns within fourteen days all but one of the seats. If a member does not attend the House for 60 days consecutively without the permission of the Presiding Officer, his seat may be declared vacant. Following reasons also can cause vacancy

he holds any office of profit declared to be unsound mind or undischargcd insolvent voluntarily acquires citizenship of another country his election is declared invalid by the court he is expelled by the House he becomes President, Vice President etc. disqualified for defection by the Presiding Officer

Sessions of Parliament The Rajya Sabha is not subject to dissolution unlike the Lok Sabha which, unless sooner dissolved, continues for five years from the date appointed for its first meeting. The Constitution provides that the President shall from time to time summon each House of Parliament to meet at such time and place as he thinks fit, but six months shall not intervene between its last sitting in one session and the date appointed from its first sitting in the next session. The President may from time to time prorogue the Parliament or either House-prorogation means the termination of a session and normally follows the adjournment sine die of the House by the Presiding Officer. A session is the period of time between the meeting of a Parliament and its prorogation. During the course of a session, either House may be adjourned for a few hours, days or weeks by the Presiding Officer for any reason. The period between the prorogation of Parliament and its reassembly in a new session is termed as a recess. Normally Parliament meets, in three sessions

in a year, namely Budget session in the months of February-March, April and May; Monsoon session in the months of July-August and Winter session in the months of November-December. Since 1994, the Budget session runs like this: after the general discussion on the Budget in the Houses is over and vote on account is passed (Art. 116) by the third week of March, the Houses are to be adjourned for a fixed period (about a month) and the committees have to consider the Demands for Grants of the assigned Ministries during one month period. Prorogation and its Effects Prorogation means the end of a session. A prorogation terminates a session; an adjournment is an interruption in the course of session. Usually prorogation follows the adjournment of the House sine die. The time-lag between the adjournment of the House sine die and its prorogation varies between 2 and 10 days. It is not necessary that both Houses should be prorogued simultaneously. The prorogation affects different categories of business pending before the House as follows: Bills Article 107(3) of the Constitution expressly provides that a Bill pending in Parliament shall not lapse by reason of the prorogation of the House. This saving also covers Bills pending before a Select or Joint Committee of the House(s). Notices of intention to move for leave to introduce Bills also do not lapse on prorogation and no fresh notice is necessary in the next session for that purpose. Motions and Resolutions On the prorogation, all pending notices except those relating to introduction of Bills as mentioned above, lapse and fresh notices must be given for the next session. This covers notices of motions, calling attention, resolutions, amendments, etc. Effect of dissolution of Lok Sabha All business pending before Lok Sabha lapses on its dissolution. However, the dissolution of LS also affects the business pending before the Rajya Sabha to a certain extent, as indicated below:

Bill pending in the Rajya Sabha which has not been passed by the Lok Sabha shall not lapse Bill which is pending in the Lok Sabha lapses Bill passed by the Lok Sabha and is pending in the Rajya Sabha lapses Under Article 108(5), a Joint Sitting of both Houses to resolve a deadlock on a Bill will be held notwithstanding the fact that dissolution of the Lok Sabha has intervened since the President has notified his intention to summon the Houses to meet in a Joint Sitting. Bills originating in the Rajya Sabha which are still pending in that House do not lapse. Bills originating in the Rajya Sabha which having been passed by the House and transmitted to the Lok Sabha and pending there lapse Bills originating in the Rajya Sabha and returned to that House by the Lok Sabha with amendments and still pending there on the date of its dissolution, lapse.

A Bill passed by the two Houses of Parliament and sent to the President for assent does not lapse. A Bill returned by the President to the Rajya Sabha for reconsideration to the Houses does not lapse if the dissolution of the Lok Sabha takes place without the Houses having reconsidered the Bill. The Indian Post Office (Amendment) Bill, 1986, as passed by the Houses of Parliament was submitted to the President for his assent. The Bill remained pending before him till the dissolution of the Eighth Lok Sabha in 1989. The President returned the Bill to the Rajya Sabha later for reconsideration of the Houses and it survived Lok Sabha dissolutions till it was withdrawn in 2002. On the dissolution of the Lok Sabha, Joint Committees become defunct except the Statutory Joint Committees. Members of the Rajya Sabha elected to serve on the Committee on Official Language which consists of members of both Houses continue to remain on that Committee notwithstanding the dissolution of the Lok Sabha only the members of the Lok Sabha on that Committee cease to be members of the Committee on the dissolution of that House. The reason for this position is that the Official Language Committee derives its authority from an Act of Parliament and the term of the members on that Committee is co-terminus with their term as members of the House.

Ques 7. Control over finances is the most important aspect of parliamentary system of government. Examine? Ans. Our Parliamentary system of Government based on Westminster model. The Constitution respects the principle no taxation without representation. Principles of the system of Parliamentary control over finance

Legislative prerogative over taxation legislative control over expenditure and executive initiative in financial matters There are specific provisions in the Constitution of India containing these principles: Article265 provides that no tax shall be levied or collected except by authority of law no expenditure can be incurred except with the authorisation of the Legislature (article 266) President shall, in respect of every financial year, cause to be laid before Parliament, Annual Financial Statement (article 112).

These provisions of our Constitution make the Government accountable to Parliament. With the emergence of Welfare State, role of Government expanded to provide social services to citizens like education, health, employment and housing. Government requires adequate financial resources to do so. Given the huge welfare role that the Government takes on and the security functions it has to perform, resources are necessarily scare, The need for budgeting arises to allocate scare resources to various Governmental activities. Ques. 8 : Presentation of Budget and its passage over the years in India has become an exercise in toto and devoid of popular will. Critically examine?

Ans. The Annual Financial Statement, laid before both the Houses of Parliament constitutes the Budget of the Union Government. President is responsible for having the Budget presented to the Lok Sabha (Art. 112). But, according to Article 77(3) the finance Minister has been made responsible by the President of India to prepare the Annual Financial Statement and pilot it through the Parliament. This statement takes into account a period of one financial year. The financial year commences on the 1st April each year. The statement embodies the estimated receipts expenditure of the Government of India for the financial year. There is no single budget for the entire country as the states have their own budgets. Even the budget of Government of India is divided into the Railway and the General Budget. Railway Budget The Budget of the Indian Railways is presented separately to Parliament and dealt with separately, although the receipts and expenditure of the Railways form part of the Consolidated Fund of India and the figures relating to them are included in the Annual Financial Statement. Stages of Budget in the Parliament In Parliament, the budget goes through five stages

presentation of budget with Finance Ministers speech general discussion, after which there is adjournment of the Houses so that the Standing Committees can scrutinize the demands for grants for a month voting on demand for grants in Lok Sabha passing of appropriation bills passing of finance bill.

The powers of Parliament in respect of enactment of budget are enshrined in the Constitution under Article 112 to 117. As per these, no demand or a grant or proposal for expenditure can be made except on the recommendation of the President. Parliament cannot increase tax though it can either reduce or abolish it. Charged expenditure is not to be subjected to Parliaments voting. Powers of Rajya Sabba is quite restricted in financial matters. Along with the Annual Financial Statement, the finance Minister submits the following 5 documents to the Parliament

key to the budget document (various definitions and the Constitutional provisions) budget at a glance( receipts and expenditure shown with various deficits and break tips) receipt budget expenditure budget memorandum explaining the process in the financial bill (impact of tax proposals on government finance)

There is a proposal that the Govt. take a look at the recommendation of Administrative Reforms Commission of 1967 which suggested changing of the financial year from April 1 to 31st March to October 1 to 30th Sept. The reason is that the Govt. can have a more realistic estimate based on the impact of Monsoon. Moreover, the financial year for the business class starts from the time of Diwali.

Presentation In India, the Budget is presented to Parliament on such date as is fixed by the President. The Budget speech of the Finance Minister is in two parts. Part A deals with general economic condition of the country while Part B relates to taxation proposals. The General Budget is usually presented on the, last working day of February i.e. about a month before the commencement of the financial year except in the year when General Election to Lok Sabha are held. In an election year, Budget may be presented twice, first to secure Vote on Account for four months conventionally and later in full. The election year budget is called interim budget. It is presented only if the elections are held in the first half of the calendar year. Since the incumbent government does not have the moral propriety to present a full budget, it presents interim budget which is nothing but the vote on account with the difference that period for which money is sanctioned by the Parliament in the interim budget is 4 months while normally the vote on account sanctions the amount for 2 months. The General Budget is presented in Lok Sabha by the Minister of Finance. He makes a speech introducing the Budget and it is only in the concluding part of his speech that the taxation proposals are made. The Annual Financial Statement is laid on the Table of Rajya Sabha at the conclusion of the speech of the Finance Minister in Lok Sabha. The Finance Bill which deals with the taxation proposals made by Government is introduced immediately after the presentation of Budget. It is accompanied by a memorandum explaining the provisions of the Bill and their effect on the finances of the Country. Vote on Account The general discussion on the Budget begins a few days after its presentation. Since Parliament will pass the budget only by mid-May, there is a need to sanction an amount to the Government to maintain itself after the new year sets in. A special provision is, therefore, made for Vote on Account by which Government obtains the Vote of Parliament for a sum sufficient to incur expenditure on various items for a part of the year (Art. 116). It is generally 2 months worth of expenditure. But during election year, the Vote on Account may be for a period exceeding two months- normally four months. Discussion The Budget is discussed in two stages in Lok Sabha. First, there is the General Discussion on the Budget as a whole. This lasts for about 4 to 5 days. Only the broad outlines of the Budget and the principles and policies underlying it are discussed at this stage. Consideration of the Demands by Standing Committees of Parliament After the first stage of General Discussion on both Railway as well as general. Budget is over, the House is adjourned for a fixed period, usually a month. During this period, the Demands for Grants of various Ministries/Departments including Railways are considered by concerned Standing Committees. These Committees submit reports to the House. The report of the Standing Committees are of persuasive nature .The report shall not suggest anything of the nature of cut motions. There are 24 such committees since 2004.

After the reports of the Standing Committees are presented to the House, the House proceeds to the discussion and voting on Demands for Grants, Ministry-wise. The time for discussion and Voting of Demands for Grants is allocated by Speaker in consultation with the Leader of the House. On the last day of the allotted period, the Speaker puts all the outstanding Demands to the Vote of the House, this device is popularly known as guillotine. Guillotine, in other words, is passing the demands for grants without discussion. It is done for want of time. Lok Sabha has the power to assent to or refuse to give assent to any Demand or even to reduce the amount of Grant sought by Government. Introduction and voting on demands is confined only to Lok Sabha. In Rajya Sabha there is only a General Discussion on the Budget. It does not vote on the Demands for Grants. Expenditure is of two types:

Charged expenditure. It includes the emoluments of the President and the salaries and allowances of the Chairman and Deputy Chairman of Rajya Sabha and the Speaker and Deputy Speaker of Lok Sabha, Judges of Supreme Court, Comptroller and Auditor General of India and certain other items specified in the Constitution of India. Discussion in Parliament on charged expenditure is permissible but such expenditure is not voted. Non-charged expenditure. It is the votable expenditure. Only so much of the amount is subject to the vote of Lok Sabha as is not a charged expenditure on the Consolidated Fund of India. The votable part is the demands for grants.

Cut Motions Motions for reduction to various Demands for Grants are made in the form of Cut Motions seeking to reduce the sums sought by Government on grounds of

economy or difference of opinion on matters of policy or to voice a grievance. Cut Motions are divided into following three categories: Disapproval of Policy cut i.e., a motion that the amount of the demand be reduced to Re. 1 representing disapproval of policy underlying the Demand. A member giving notice of such a Cut Motion should indicate in precise terms, the particulars of the policy which he proposes to discuss. It is open to the member to advocate an alternative Policy. Economy cut i.e., a motion that the amount of the Demand be reduced by a specific amount representing the economy that can be effected. And Token cut i.e., a motion that the amount of the Demand be reduced by Rs. 100 in order to express a specific grievance. It is generally the opposition party member who may seek to move a cut motion. Admissibility of the cut motion is entirely the discretion of the Speaker. There is speculation as to what will happen if the cut motion is moved. It is the consensus opinion that in such a situation, the Government needs to show that it has majority by bringing forward a confidence motion under Rule 184 of the Lok Sabha.

Appropriation Bill After the Voting on Demands for Grants have been completed by late April or early May, Government introduces the Appropriation Bill. The Appropriation Bill is intended to give authority to Government to incur expenditure from the Consolidated Fund of India. The appropriation Bill will appropriate the sums that the Lok Sabha granted by voting demands for grants. It also includes the charge expenditure. The procedure for passing this Bill is the same as in the case of other money BIlls. Ques. 9 : Budget is an important tool of legislative control over the executive'. Examine the statement in light of the working of Indian Parliament over the years? Ans. The Finance Bill (one that contains taxation proposals and is presented as a part of the budget) is introduced in Lok Sabha immediate after the presentation of the General Budget. Certain provisions in the Bill relating to levy and collection of fresh duties or variations in the existing duties come into effect immediately on the expiry of the day on which the Bill introduced by virtue of a declaration under the Provisional Collection of Taxes Act. Parliament shall pass the Finance Bill within 75 days of its introduction. Budget is an important tool of legislative control over the executive. It is also an instrument of economic and social policies in line with the five year plan. Financial Bills They are contained in Art. 117. There are two types: Financial Bill which is a Bill in which there are provisions related to a Money Bill but also those of an ordinary Bill. It has two features in common with the Money Bill:

Presidents prior recommendations is necessary and It can be introduced only in the Lok Sabha

Other provisions are similar to the ordinary Bill. Financial Bill is an ordinary Bill with the difference that, when it is passed, it entails expenditure from the CFI. As far as procedure is concerned, it is passed like a Money Bill but before the commencement of the second reading (Consideration, stage two) in Both the Houses, the recommendation of the President is necessary. Supplementary, Additional, Token, Excess and Exceptional Grants Art. 115 contains provisions related to Supplementary, additional, and excess grants

Supplementary grant may be made by the Parliament if the amount authorised in the budget passed originally for a particular service for the current financial year is found to be insufficient Additional grant may be made by Parliament for expenditure on some new service not contemplated in the annual financial statement for that year Excess grant is made by Parliament if any money has been spent on any service during a financial year in excess of the amount granted for that service and for that year. In other words, it is a grant to retrospectively authorise excess of expenditure committed by the

executive. CAG detects it while auditing the appropriation accounts and is satisfied that it is justified and the PAC, on the basis of he CAG report, recommends such retrospective regularization. Token grant is one where the Department / ministry has the money to spend on a new service. The availability of money is by way of reappropriation- spending money sanctioned for one head on another head within the same ministry with the permission of the same Ministry. But it seeks a token sum of Rs. 1 from Lok Sabha. Exceptional grant is seeking money for a service that is not part of the current service of any financial year.

Budget of a State under Presidents Rule Budget of a State under Presidents rule is presented to Lok Sabha. The procedure followed in regard to the Budget of the Union Government is followed in the case of State Budget also with such variations or modifications, as the Speaker may make. Money Bill (Art. 110) Bills which exclusively contain provisions for

the imposition, abolition, remission, alteration or regulation of any tax; the regulation of the borrowing of money or the giving of any guarantee by the Government of India the custody of the Consolidated Fund or the Contingency Fund of India, the payment of moneys into or the withdrawal of moneys from any such Fund the appropriation of moneys out of the Consolidated Fund of India; the declaring of any expenditure to be expenditure charged on the Consolidated Fund of India or the increasing of the amount of any such expenditure; the receipt of money on account of the Consolidated Fund of India or the public account of India or the custody or issue of such money or the audit of the accounts of the Union or of a State; or any matter incidental to any of the matters specified above.

If any question arises whether a Bill is a Money Bill or not, the decision of the Speaker of the House of the People thereon shall be final. There shall be endorsed on every Money Bill when it is transmitted to the Council of States under Article 109, and when it is presented to the President for assent under Article 111, the certificate of the Speaker of the House of the People signed by him that it is a Money Bill. Money Bill can be introduced only in Lok Sabha and on Presidents recommendation. Rajya Sabha cannot make amendments in a Money Bill passed by Lok Sabha and transmitted to it. It can, however, recommend amendments in a money Bill, but must return all Money Bills to Lok Sabha within fourteen days from the date of their receipt. It is open to Lok Sabha to accept or reject any or all of the recommendations of Rajya Sabha with regard to a Money Bill. If a Money Bill passed by Lok Sabha and transmitted to Rajya Sabha is not returned to Lok Sabha within the said period of fourteen days, it is deemed to have been passed by both Houses at the expiration

of the said period in the form in which it was passed by Lok Sabha. President can assent to or reject a Money Bill but can not return it for repassage. Consolidated Fund of India The fund constituted under Article 266 (1) of the Constitution of India into which all receipts, revenues and loans flow. All expenditure from the CFI is by appropriation Bill. Public Account of India It includes those moneys where the Government acts as a banker. For example, PF, small savings etc Article 266 (2) of the Constitution of India. Thes funds do not belong to the government. They have to be paid back at some time to their rightful owners. Therefore, expenditure from it is not required to be approved by the Parliament. Some dedicated funds are also part of PAI. For example : Reserve Funds bearing interest (railway finds, telecommunication funds) and Reserve Funds not bearing an interest (the Central Road Fund, Sugar Development Fund). Contingency Fund of India Parliament has by law established a Contingency Fund placed at the disposal of the President to enable advances to be made by him out of it for the purpose of meeting unforeseen expenditure pending authorisation of such expenditure by Parliament by Law (Art. 267). Earlier contingency fund was of Rs 50 Crore but was recently raised to Rs 500 crore by parliament. Parliamentary Control Over Public Finance The instruments of control that the Parliament has over the public finance are the following:

Art.265 says that no tax can be levied and collected except by the authority of law Art.266 says that money can be drawn from the Consolidated Fund of India only by an Appropriation Bill passed by the Parliament Parliament, under Art. 292, can regulate the borrowing power of the Executive on the security of the Consolidated Fund of India Based on Art. 292 partly, FRBM Act was made in 2003 to bring public finance under parliamentary accountability- setting limits to borrowing and the FM having to explain to the Parliament on a quarterly basis the budgetary trends Cut motions available to Lok Sabha members during the passage of demands for grants can lead to compelling the Government to take up or drop any policy. However, cut motions have never been passed so far Budget under Art. 112 is a socio-economic statement Parliament can alter the priorities as it has to be approved by it to take effect other than the Constitutional provisions ,mentioned above, the following are the statutory and other developments that make the Parliamentary control over government finance effective. Outcome budget is being presented from 2004-05 onwards where for a given outlay the expected impact in socio-economic terms is quantified and declared. Parliament can hold the government answerable for the achievement of the outcomes

Similarly, 54 gender budget cells are set up by the ministries in the Union Government (2008). Parliament can seek explanation from the government as to how far it has progressed in adopting a gender perspective in public policy. Parliamentary financial committees- PAC, Estimates Committee and CPU exercise control on the executive by ensuring that the moneys spent are not wasted and are well targeted Under parliamentary pressure, government is presenting the Tax Expenditure Statement since 2006 showing revenue foregone through tax breaks. In course of time, there will be greater parliamentary control on such losses of revenue through exemptions.

Thus, the control that the Parliament can exercise on public finance is enormous. Some controls are provided in the Constitution (Art. 112, 265, 266, 292 etc). Others are derived from statute and rules as shown above. Parliamentary control over the executive Art.75.3 says that the Council of Ministers enjoys power till it has support in the Lok Sabha. Parliament controls the executive through a variety of means and instruments in order to enable people to have responsible and accountable government. It takes place through the following instruments: Motions like confidence motion, adjournment motion etc. Questions of various types Ques. 10 : Bring out the difference between Adjournment Motion and No-confidence Motion? Ans. An adjournment motion is moved to discuss a matter of urgent public importance and is available only in Lok Sabha and not in Rajya Sabha as it means severe criticism of the Government and is used to express dissatisfaction with the Government policies. Speakers consent is necessary for moving the motion. After the motion is moved, it can be proceeded with only if 50 members endorse it. Speaker may not admit the motion in case there are other means of raising the matter for discussion. The motion shall not deal with any matter which is under adjudication by a court of law. No-confidence Motion Art. 75(3) says that the Council of Ministers is collectively responsible to the Lok Sabha. The Council of Ministers is in power only as long as it enjoys the numerical majority of the Lok Sabha. The opposition parties have an instrument in the form of no confidence motion to remove the Government. The procedure for the NCM is not a part of the Constitution but is given in the Rules of Procedure and Conduct of Business of Lok Sabha. Rule 198 says that an NCM may be moved subject to the following conditions: Leave to make the motion shall be asked for by the member when called by the Speaker; Speaker reads the motion to the House and if 50 members support it, it should be taken up on such day, not being more than ten days from the date on which the leave is asked for as he may appoint.

If leave is granted, the Speaker may allot time for the discussion of the motion. After members speak for and against the motion, Prime Minister replies before vote is taken. If the motion carries majority vote, the Government has to resign. Following needs to be noted regarding the NCM

No grounds need be mentioned to move the motion No conditions of admissibility are mentioned.

Motion of Confidence in the Council of Ministers An essential tenet of the Westminster system is that the Government must he collectively responsible to the representative House. In India, the doctrine of collective responsibility of the Union Executive to the House of the People is specifically enshrined in the Constitution (Art. 75.3) Loss of confidence of the popular House requires the Government, to resign and facilitate installation of an alternative Government, if possible or the President dissolves the LS and general election to LS is held. The usual procedure to express want of confidence in the Council of Ministers is through a motion of no confidence under Rule 198 of the Rules of Procedure and Conduct of Business in Lok Sabha. The device of confidence motion is of recent origin. There is no rule in the Rules of Procedure relating to Motion of Confidence in the Council of Ministers. The need of raising debate through such a motion arose in the late seventies with the advent of minority Governments and later, by mid-1990s, formation of coalition Governments as a result of hung Parliaments. In the absence of any specific rule in this regard, such Motions of Confidence have been entertained under the category of motions stipulated in Rule 184 which are meant for raising discussions on matters of public interest in Lok Sabha. In the case of a Confidence Motion, there is no requirement for seeking leave of the House. The one line notice of motion under Rule 184 that This House expresses its confidence in the Council of Ministers is given by the Prime Minister at the behest of a Presidential direction. When admitted by the Speaker, it is bullet med. The date and time for its discussion is fixed in consultation with the Business Advisory Committee. Confidence Motion and No-Confidence Motion have the same purpose: the government has to demonstrate its majority in Lok Sabha. The former is a Government initiative when called upon to do so by the President and the latter is moved by the opposition party member. The notice of the first ever Motion of Confidence was given by the then Prime Minister Ch Charan Singh in 1979. This motion could not be moved as Ch. Charan Singh tendered the resignation. The first Motion of Confidence was moved by Shri V.P. Singh, the then Prime Minister in December, 1989 in the Lok Sabha which was adopted by the House by the voice vote on the same day. Since their ten Motions of Confidence have been moved (2008). Censure Motion Censure motion can be moved only in Lok Sabha under Rule 184. Speaker can disallow a censure motion. Grounds need to be mentioned unlike the NCM. It can be moved against an individual minister or a group of ministers unlike the NCM. The censured ministers need not resign, unlike the NCM. Under Rule 184, voting takes place at the end of the debate. Rule .184 is a way of parliament enforcing accountability of the executive.

Rule 184 and Rule 193 in Lok Sabha Rule 184 is a censure motion as the debate ends with vote. It is classified as No Day Yet Named Motion in the Rules. It relates to any matter of general public interest. Speaker decides the admissibility of the motion. If the Government loses for any reason, it has to come forward with a confidence motion to establish its numerical majority. In 2002, in relation to the developments of Gujarat, opposition moved a censure motion under Rule 184 and the Deputy Speaker allowed the same. The Government won the motion. Rule 193 allows a short duration discussion and is not followed by vote. The matter should be one of urgent public importance and the notice of the member to raise the issue should be supported by at least two other members. In the Lok Sabha in the winter session Nov-Dec 2007, four Short Duration discussions under Rule 193 were held on the following issues:

Proposal to set up Special Economic Zone in Nandigram, West Bengal and consequent large scale violence. Indo-US nuclear agreement Need for harmonious functioning of three organs of the State i.e. Legislature, judiciary and executive. Internal Security

In the Rajya Sabha, debate under Rules 167 and 170 ends in voting while Rule 191 does not need voting. However, censure motion of the Lok Sabha type does not exist in Rajya Sabha. Question Hour The first hour of every sitting of Lok Sabha is called the Question hour. Questions are of three types Starred Unstarred and Short Notice A Starred Question is one to which a member wants an oral answer in the House and which is distinguished by an asterisk mark. To a starred question, members can put supplementaries because the Minister is orally answering the question. An unstarred Question is one which is answered in writing and no supplementary questions can be asked. Minimum period of notice for starred / unstarred question is 10 clear days (thats, excluding holidays). Short Notice Questions They do not require the normal period of notice of 10 days. They relate to matters of urgent public importance. A Short Notice Question may only be admitted if permitted by the Speaker. A Short Notice Question is taken tip for answer immediately after the Question Hour. A question may also be addressed to a Private Member provided that the subject matter of the question relates to some Bill, Resolution or other matter connected with the business of the House for which that Member is responsible.

Half-an-Hour Discussion A Half-an-Hour Discussion can be raised on a matter of sufficient public importance which has been the subject of a recent question. During the discussion, the member makes a short statement and not more than four members are permitted to ask a question each for further clarity. Thereafter, the Minister concerned replies. There is no formal motion before the House nor voting. Rule 377 Under Rule 377 of the Rules of Procedure and Conduct of Business in the Lok Sabha members are allowed to raise matters which are not points of order or which cannot be raised under any other Rule. Legislation or How a Bill becomes an Act A Bill is the draft of a legislative proposal. It has to pass through three stages and receive the assent of the President before it becomes an Act of Parliament. It will come into effect after it has been notified by the Government. The Bills initiated by Ministers are called Government Bills and those introduced by Members who are not Ministers, are known as Private Members Bills. Depending on their contents, Bills may further be classified broadly into

Ordinary Bills Constitution Amendment Bills Money Bills Financial Bills A and B Other Bills where the procedure for passing of the Bill may be marginally different- for example, requiring the prior assent of the President (Art.3) Process of passage of a Bill in each House is as follows.

First Reading The legislative process starts with the introduction of a Bill in either House of Parliament--Lok Sabha or Rajya Sabha. A Bill can be introduced either by a Minister or by a private member. In the former case it is known as a Government Bill and in the later case it is known as a Private Members Bill. It is necessary for a member-in-charge of the Bill to ask for leave to introduce the Bill A Minister has to give notice of days and a private member 30 days for seeking leave of the House for introduction. If leave is granted by the House, the Bill is introduced. This stage is known as the First Reading of the Bill. If the motion for leave to introduce a Bill is opposed, the Speaker/Chairman may, in his discretion, allow brief explanatory statements to be made by the member who opposes the motion and the member-in-charge who moved the motion. Where a motion for leave to introduce a Bill is opposed on the ground that the Bill initiates legislation outside the legislative competence of the House, the Speaker/Chairman may permit a full discussion. Thereafter the question is put to the vote of the House. However, the motion for leave to introduce a Finance Bill or an Appropriation Bill is forthwith put to the vote of the House.

Publication in Gazette After a Bill has been introduced, it is published in the Official Gazette. Even before introduction, a Bill may, with the permission of the Presiding Officer, be published in the Gazette. In such cases, leave to introduce the Bill in the House is no asked for and the Bill is straightaway introduced. Reference of Bill to Standing Committee After a Bill has been, introduced, Presiding Officer of the concerned House can refer the Bill to concerned Standing Committee for examination and make report. If a Bill is referred to Committee, the Committee shall consider the general principles and clauses of the Bill referred to them and make report. The Committee can also take expert opinion or the public opinion who are interested in the measure) After the 131 has thus been considered, the Committee submits its report to the House: The report of the Committee has persuasive value. Second Reading or Consideration The Second Reading consists of consideration of the Bill which is in two stages. First Stage: The first stage consists of general discussion on the Bill as a whole when the principle underlying the Bill is discussed. At this stage it is open to the house to

refer the Bill to a Select Committee of the House or a Joint Committee of the two Houses or to circulate it for the purpose of eliciting opinion or to straightaway take it into consideration.

If a Bill is referred to a Select/Joint Committee, the Committee considers the Bill clause-byclause just as the House does. Amendments can be moved to the various clauses by members of the Committee. The Committee can also take evidence of associations, public bodies or experts who are interested in the measure. After the Bill has thus been considered, the Committee submits its report to the House which considers the Bill again as reported by the Committee. If a Bill is circulated for the purpose of eliciting public opinion thereon, such opinions are obtained through the Governments of the States and Union Territories. Opinions so received are laid on the Table of the House and the next motion in regard to the Bill must be for its reference to a Select/Joint Committee. It is not ordinarily permissible at this stage to move motion for consideration of the Bill. Second Stage: The second stage of the Second Reading consists of clause-by-clause consideration of the Bill as introduced or as reported by Select/Joint Committee. Discussions take place on each clause of the Bill and amendments to clauses can be moved at this stage. Third Reading or voting Thereafter, the member-in-charge can move that the Bill be passed. This stage is known as the Third Reading of the Bill or voting. At this stage debate is confined to arguments either in support or rejection of the Bill without referring to the details thereof further than that are absolutely necessary. Only formal, verbal or consequential amendments are allowed to be moved at this stage. In passing an ordinary Bill, a simple-majority of members present and voting is necessary. But in the case of a Bill to amend the Constitution, called special majority is necessary.

Bill in the other House After the Bill is passed by one House it is sent to the other House for concurrence with a message to that effect. Consideration of the Bill at a Joint Sitting (Art.108) If a Bill passed by one House is

rejected by the other House, or the Houses have finally disagreed as to the amendments to be made in the Bill, or more than six months elapse from the date of the receipt of the Bill by the other house without the Bill being passed by it

The President may call a joint sitting of the two houses to resolve the deadlock. If, at the joint sitting of the Houses, the Bill is passed by a majority of the total number of members of both the Houses present and voting, with the amendments, if any, accepted by them, the Bill is deemed to have been passed by both the Houses. Assent of the President When a Bill is passed by both Houses, the Bill is sent for the assent of the President. The President may give his assent or withhold his assent to a Bill. The President may also return the Bill (except a Money Bill) with his recommendations to the Houses for reconsideration, and if the Houses pass the Bill again with or without amendments by a simple majority, the President cannot withhold his assent to the Bill (suspensive veto). For example, the Parliament (Prevention of Disqualification) Amendment Bill, 2006 that is popularly refered to as the Office of Profit Bill was returned for repassage by the President in 2006 after it was passed by the parliament. When repassed and sent again, the President gave his assent to the Office of Profit Bill. The President, however, is bound to give his assent to a Constitution Amendment Bill passed by the Houses of Parliament by the requisite special majority and, where necessary, ratified by the States (24th Amendment Act 1971). Ques. 11 : Joint sitting of the Parliament ensures smooth passages of bills based on consensus. Discuss? Ans. The Constitution of India envisages, a mechanism for resolving disagreement between the two Houses in respect of a Bill, other than a Money Bill or a Constitution Amendment Bill. In case of a Money Bill, the powers of the Rajya Sabha are limited to retaining or delaying the Bill passed by the Lok Sabha for a period of fourteen days only and recommending an amendment or amendments in the Bill which may or may not be accepted by the Lok Sabha. In case of a Constitution Amendment Bill, if both Houses do not pass such a Bill in identical terms, in accordance with Article 36, there is an end of that Bill. It will have to be reintroduced. When any other Bill is deadlocked, the President may, unless the Bill has lapsed by reason of dissolution of the Lok Sabha, notify to the Houses by message, if they are sitting, or by public notification, if they are not sitting, his intention to summon them to meet in a joint sitting for the purpose of deliberating and voting on the Bill.

The Speaker and in his absence the Deputy Speaker of the Lok Sabha or if he is also absent, the Deputy Chairman of the Rajya Sabha or if, he/she too is absent,such other person as may be determined by the members present at the sitting, presides over the joint sifting. Chairman can not preside as he is not a member of Rajya Sabha. The procedure of the Lok Sabha applies at a joint sitting. The quorum to constitute a joint sitting is one-tenth of the total number of members of the two Houses. If at a joint sitting, the Bill referred to it, with such amendments, if any, as are agree in the joint sitting, is passed by a majority of the total number of members of both Houses present and voting, it is deemed to have been passed by both Houses. At a joint sitting no amendment can be proposed to the Bill, other than such amendments, if any, as become necessary by the delay in its passage and such other amendments as relate to matters with respect to which the Houses have no agreed in their individual capacity. The decision of the presiding officer as to the admissibility of amendments is final. At a joint sitting, the Speaker or the person presiding as such shall not vote in the first instance, but shall have and exercise a casting vote in the case of equality of votes. Joint sittings held so far So far joint sittings have been held thrice in 1961, 1978 and 2002.

The first joint sitting was held in 1961 to consider amendments to the Dowry Prohibition Bill 1959. Members of both Houses were united in their support for the Bill, but differed over specific nuances. The second joint sitting was held in 1978, after the Rajya Sabha rejected the Banking Service Commission (Repeat) Ordinance, 1977. The Bill was passed. The third joint sitting was held to make POTO into an Act in 2002-Prevention of Terrorism Bill. It became necessary as the Bill was rejected by the Rajya Sabha. The joint sitting, sanctioned under Article 108 of the Constitution, was intended by the Constitution-makers as a way to resolve disagreements between the two Houses in matters of legislation. It gives another chance for the two Houses to reconcile their differences in national interest.

Ques. 12 : With the work of Parliament increasingly becoming move technical and voluminous, Parliamentary committees have became significant. Discuss? Ans. The Parliament work is increasingly becoming more technical and voluminous. It cannot, therefore, give close consideration to all the legislative and other matters before it. It is one of the reasons for setting up Parliamentary committees that do some of the parliamentary work. Committees also provide greater focus and expertise to the parliamentary work. Parliamentary Committees are of two kinds: Ad hoc Committees and Standing Committees Ad hoc Committees Ad hoc Committees are appointed for a specific purpose and they cease to exist when they finish the task assigned to them and submit report. Ad hoc Committees are of two types

committees which are constituted from time to time to inquire into and report on specific subjects, (for example, Committee on Members of Parliament Local Area Development Scheme, Joint Committee on Bofors Contracts, Joint Committee to enquire into irregularities in securities and banking transactions, Joint Committee on Stock Market Scam etc.) Select or Joint Committees on Bills which are appointed to consider and report on a particular Bill. These Committees are distinguishable from the other ad hoc committees in as much as they are concerned with Bills and the procedure to be followed by them as laid down in the Rules of Procedure and Directions by the Speaker/Chairman.

Standing Committees Apart from the Ad hoc committees, each House of Parliament has Standing Committees like the Business Advisory Committee, the Committee on Petitions, the Committee of Privileges and the Rules Committee etc. They are elected or appointed every year or periodically and their work goes on, more or less, on a continuous basis. Among the Standing Committees, the three Financial CommitteesCommittees on Estimates, Public Accounts and Public Undertakingsconstitute a distinct group as they keep vigil over Government expenditure and performance. While members of the Rajya Sabha are associated with Committees on Public Accounts and Public Undertakings, the members of the Committee on Estimates are drawn entirely from the Lok Sabha. Other Standing Committees in each House, divided in terms of their functions, are i) Committees to Inquire

Committee on Petitions examines petitions on bills and on matters of general public interest and also entertains representations on matters concerning subjects in the Union List; and Committee of Privileges examines any question of privilege referred to it by the House or Speaker/Chairman Committee on Government Assurances keeps track of all the assurances, promises, undertakings, etc., given by Ministers in the House and pursues them till they are implemented Committee on Subordinate Legislation scrutinises and reports to the House whether the power to make regulations, rules, sub-rules, bye-laws, etc., conferred by the Constitution or Statutes is being properly exercised by the delegated authorities Business Advisory Committee recommends allocation of time for items of Government and other business to be brought before the Houses Committee on Private Members Bills and Resolutions of the Lok Sabha classifies and allocates time to Bills introduced by private members, recommends allocation of time for discussion on private members resolutions and examines Constitution amendment bills

ii) Committees to Scrutinise

iii) Committees relating to the day-to-day business of the House


before their introduction by private members in the Lok Sabha. The Rajya Sabha does not have such a committee.

Rules Committee considers matters of procedure and conduct of business in the House and recommends amendments or additions to the Rules Committee on Absence of Members from the Sittings of the House of the Lok Sabha considers all applications from members for leave or absence from sittings of the House. There is no such Committee in the Rajya Sabha. Committee on the Welfare of Scheduled Castes and Scheduled Tribes, on which members from both Houses serve, considers all matters relating to the welfare of Scheduled Castes and Scheduled Tribes which come within the purview of the Union Government and keeps a watch whether constitutional safeguards in respect of these classes are properly implemented General Purposes Committee considers and advises Speaker/ Chairman on matters concerning affairs of the House, which do not appropriately fall within the purview of any other Parliamentary Committee House Committee deals with residential accommodation and other amenities for members

(iv) Committees concerned with the provision of facilities to members

v) Joint Committee on Salaries and Allowances of Members of Parliament, constituted under the Salary, Allowances and Pension of Members of Parliament Act, 1954, apart from framing rules for regulating payment of salary, allowances and pension to Members of Parliament, also frames rules in respect of amenities like medical, housing, telephone, postal, constituency and secretarial facility vi) Joint Committee on Offices of Profit examines the composition and character of committees and other bodies appointed by the Central and State governments and Union Territories Administrations and recommends what offices ought to or ought not to disqualify a person from being chosen as a member of either House of Parliament vii) The Library Committee consisting of members from both Houses, considers matters concerning the Library of Parliament viii) In 1997, a Committee on Empowerment of Women with members from both the Houses was constituted with a view to securing, among other things, status, dignity and equality for women in all fields ix) In 1997, the Ethics Committee of the Rajya Sabha was constituted. The Ethics Committee of the Lok Sabha was constituted in 2000. Watchdog committees Of special importance is a class of Committees which act as Parliaments Watch Dogs over the executive. These are the

Committees on Subordinate Legislation Committee on Government Assurances

Committee on Estimates Committee on Public Accounts Committee on Public Undertakings and Departmentally Related Standing Committees (DRSCs).

The Committee on Estimates, the Committee on Public Accounts and the committee on Public Undertakings and DRSCs play an important role in exercising a check over governmental expenditure and Policy formulation. Select and Joint Committees After a Bill is introduced in the House, it is open to that House to refer it to a Select Committee of the House or a Joint Committee of the two Houses. A motion has to be moved and adopted to this effect in the House. The decision is conveyed to the other House requesting them to nominate members of the other House to serve on the Committee. The Select or Joint Committee considers the Bill clause by clause. Amendments can be made by the Committee. After the Bill has thus been considered the Committee submits its report to the House. The report of the committee is not binding but only guides the legislative process. Ques. 13 : Departmentally standing committees have made the parliamentary control move effective and focussed particularly in financial matters. Examine? Ans. To make the parliamentary control more effective and focused, particularly in financial matters and public policy, and to make the Executive more accountable to the Parliament, Departmentally related Standing Committees were set up first in 1993. They cover under their jurisdiction all the Ministries/Departments of the Union Government each committee being given one or more ministries. Similar system of Departmentally Related Select Committees was in existence in the United Kingdom unlike the Consultative Committees attached to various ministries which are informal in nature and are presided over by the Ministers concerned, the meetings of the Standing Committees under consideration are presided over by private members. There are 24 DRSCs with 31 members each- 21 from Lok Sabha and 10 from Rajya Sabha to be nominated by the Speaker and Chairman respectively. The term of the Members of the Committees shall not exceed one year. The functions of these Committees would broadly include:

consideration of Demands for Grants. examination of Bills referred to them Presiding Officers consideration of Annual Reports. consideration of national basic long term policy documents tabled in the Parliament and referred to the Committee by the Chairman or the Speaker.

It is agreed that there should be a separate Committee for the Ministry of Labour and Welfare and two separate Committees for the Ministry of Commerce and Industry. It may be noted that a Minister is barred from being nominated as a member of a Standing,

Committee, and if a member after his nomination to the Committee becomes a Minister, he ceases to be a member of the Committee from. the date of his appointment. The Reports of the Committees have persuasive value. In respect of reports Demands for Grants and other subjects, the Ministry or the Department concerned is required take action on the recommendations and observations contained in the report and Action Taken Reports are presented to House. The biggest achievement of these Standing Committees would be that the Demands for Grants of most of all Ministries/Departments of the Government would be scrutinised by members of Parliament. It will ensure greater participation of Members. Previously, because of paucity of time, the Parliament was able to discuss the Demands for Grants of only a few Ministries every year and the rest were guillotined. Now, after the general discussion on the Budget is over, the Parliament shall adjourn for a fixed period about a month and the Committees shall consider the Demands for Grants during this recess. The Demands for Grants shall thereafter be considered by the Lok Sabha in the light of the Reports of these Committees. The Standing Committees system is the latest innovation in the ever evolving process of Parliamentary surveillance over the Executive to ensure its accountability to the common man. Criticism is that the reports of the committees are not being paid due attention. Guillotine process still continues. Ques. 14 : Bring out the difference between Public Account Committee and Joint Parliamentary Committee and which according to you is move effective in enforcing probity in executive functioning and why? Ans. The Committee on Public Accounts is constituted by Parliament each year for examination of accounts showing the appropriation of sums granted by Parliament for expenditure of Government of India. It is the oldest committee being in existence from 1921. The Committee consists of not more than 22 members comprising 15 members elected by Lok Sabha every year from amongst its members according to the principle of proportional representation by means of single transferable vote and not more than 7 members of Rajya Sabha elected by that House in like manner are associated with the Committee. The Chairman is appointed by the Speaker from amongst its members. The Speaker, for the first time, appointed a member of the Opposition as the Chairman of the Committee for 1967-68. This practice has been continued since then. A Minister is not eligible to be elected as a member of the Committee. If a member after his election to the Committee is appointed a Minister, he ceases to be a member of the Committee from the date of such appointment. Functions The Appropriation Accounts relating to the Railways, Defence Services, P&T Department and other Civil Ministries of the Government of India and Reports of the Comptroller and Auditor General of India mainly form the basis of the deliberation of the Committee. In scrutinising the Appropriation Accounts and the Reports of the Comptroller and Auditor General, it is the duty of the Committee to satisfy itself:

that the money shown in the accounts as having been disbursed were legally available for and, applicable to the service or purpose to which they have been granted that the expenditure conforms to the authority which governs it; and that every re-appropriation has been made in accordance with the provisions made in this behalf under rules framed by competent authority.

One of the duties of the Committee is to ascertain that money granted by Parliament has been spent by Government within the scope of the demand. If ally money has been spent on a service in excess of the amount granted by the House for the purpose, the Committee examines with reference to the facts and circumstances leading to such an excess and makes such recommendations as it may deem fit. If it recommends retrospective authorization by Parliament of such: grants, they are called excess grants (Art. 115). The functions of the Committee extend however, beyond, the formality of - expenditure to its wisdom, faithfulness and economy. The Committee thus examines cases involving losses, nugatory expenditure and financial irregularities. While scrutinising the Reports of the Comptroller and Auditor General on Revenue Receipts, the Committee examines various aspects of Governments tax administration. The Committee thus examines cases involving under-assessments, tax-evasion, non-levy of duties, misclassifications etc., identifies the loopholes in the taxation laws and procedures and makes recommendations in order to check leakage of revenue. The Comptroller and Auditor General is the friend, philosopher and guide of the Committee, lie attends the sittings of the Committee and assists it in its labours. Since the Committee became a Parliamentary Committee under the control of the Speaker 1950, it has presented more than 1250 Reports till today. PAC is one of the watchdog committees of the Parliament. Government takes action on the recommendations of the Committee. The Committee watches the implementation of its recommendations. In case where the Government has reasons to disagree with a recommendation of the Committee, the latter may, if it thinks fit, present a further report after considering the view of the Government. Committee on Estimates The Estimates Committee, constituted for the first time in 1950, is a Parliamentary Committee consisting of 30 Members, elected every year by the Lok Sabha from amongst its Members. The Chairman of the Committee is appointed by the Speaker from amongst its members. A Minister cannot be elected as a member of the Committee and if a member after his election to the committee is appointed a Minister, deceases to be a member of the Committee from the date of such appointment. The term of office of the Committee is one year. Functions The functions of the Estimates Committee are to report what economies, improvements in organisation, efficiency or administrative reform, consistent with the policy underlying the estimates may be effected; to suggest alternative policies in order to bring about efficiency and economy administration; to examine whether the money is well laid out within the limits of the policy implied in the

estimates; and to suggest the form in which the estimates shall be presented to Parliament. The Committee does not exercise its functions in relation to such Public Undertakings as are allotted to the Committee on Public Undertakings. It is called the continuous economy committee as it reports throughout the year on what savings can be made with what administrative reforms. Committee on Public Undertakings Constitution The Committee on Public Undertakings is a Parliamentary Committee consisting of 22 Members, fifteen elected by the Lok Sabha and seven by the Rajya Sabha from amongst their Members according to the principle of proportional representation by means of a single transferable vote. The Chairman is appointed by the Speaker from amongst the Members of the Committee. A Minister is not eligible to become a Member of the Committee. If a Member after his election to the Committee is appointed a Minister, he ceases to be a Member of the Committee from the date of such appointment. The term of the Committee does not exceed one year. Functions The functions of the Committee on Public Undertakings are

to examine the reports and accounts of Public Undertakings specified in the Fourth Schedule to the Rules of Procedure and Conduct of Business in Lok Shha to examine the reports, if any, of the Comptroller and Auditor General of India on the Public Undertakings to examine, in the context of the autonomy and efficiency of the Public Undertakings whether the affairs of the Public Undertakings are being managed in accordance with sound business principles and prudent commercial practices.

The Committee selects from time to time for examination such Public Undertakings or such subjects as they may deem fit and as fall within their terms of reference. Ques. 15 : Parliamentary privileges have become a sine quo rias for suppressing the freedom of speech'. Examine the statement with suitable examples? Ans. Privilege is an exemption from the general law. It is a special right. Parliamentary privilege consists of the rights and immunities which the two houses of Parliament and their members possess to enable them to carryout their parliamentary functions effectively. Without this protection members would be handicapped in performing their parliamentary duties, and the authority of Parliament would be correspondingly diminished. Members enjoy some of the privileges and immunities while the session is on and other all the time while they remain as members Privileges are contained in Articles 105 and 194- for Parliament and state legislatures respectively. There are two related expressions in this context- Breach of Privilege and Contempt of the House The difference between the two is the following:

When any of the privileges either of the Members individually or of the house in its collective capacity are disregarded or attacked by any individual or authority, the offence is called a breach of privilege. Any obstruction or impediment put before Houses or its Members in due discharge of their duties, or which have a tendency of producing such result, may amount to contempt of the House. Thus, breach refers to specific privilege being breached. Contempt is a general expression. Both are by and large synonymous Constitutional Provisions The Constitution specifies some of the privileges in Art. 105. They are :

freedom of speech in Parliament where the restrictions under Art. 19.2 do not apply but the Parliament may prescribe its own rules. (Art. 105.2) However, members cannot discuss the personal conduct of a judge of Supreme Court or High Court unless proceedings for impeachment are being held (Art.121) immunity to a member from any proceedings in any court in respect of anything said or any vote given by him in Parliament or any committee thereof Courts are prohibited from inquiring into the validity of any proceedings in Parliament on the ground of an alleged irregularity of procedure (Art.122)

Statutory Provisions Apart from the privileges specified in the Constitution (Art.105), the Code of Civil Procedure, 1908, provides for freedom from arrest and detention of members under civil process during the continuance of the meeting of the House or of a committee thereof and forty days before its Commencement and forty days after its conclusion. However, the protection does not apply in criminal and preventive detention cases. Privileges based on Rules of Procedure and Precedents The House has a right to receive immediate information of the arrest, detention, conviction, imprisonment and release of a member on a criminal charge or for a criminal-offence. Members or officers of the House cannot be compelled to give evidence or to produce documents in courts of law, relating to the proceedings of the House without the permission of the House. Consequential powers of the House Each House enjoys powers necessary for the Protection of its privileges and immunities. These powers are

to commit persons, whether they are members or not, for breach of privilege or contempt of the House to regulate its procedure and the conduct of its business to prohibit the publication of its debates and proceedings and to exclude strangers.

Penal Powers of the House The House has the power to determine as to what constitutes breach of privilege and contempt. A person found guilty of breach of privilege or contempt of the House may be punished either by imprisonment, or by admonition (warning) or reprimand. Two other punishments may also be awarded to the members for contempt, namely, suspension and expulsion from the House. In 2005, Parliament expelled it members-10 from the Lok Sabha and one from the Rajya Sabha whose conduct was found to be unethical and unbecoming of Members of Parliament. For the first time in the annals of Parliament, the membership of the 11 MPs was terminated by voice vote, following the sting operation on the cash or questions scandal. The Rajya Sabha agreed with the recommendation of its Ethics Committee while the Lok Sabha endorsed the report of the Pawan Kumar Bansal Committee set up to go into the allegations. Keshav Singh case 1964 Keshav Singh had published a pamphlet maligning a member of the State Legislative Assembly. The House found him guilty of contempt and sentenced him to prison for seven days. He challenged this order before the High Court, which granted him interim bail. The House declared that the judges who issued interim orders were themselves guilty of contempt of the House and liable to be punished. The judges moved petitions before the High Court, which sat in a full bench and stayed the orders of the House. As this confrontation seemed to be spiraling out of control, the Union government requested the President to refer the matter to the Supreme Court under Art.143. The key argument before the Supreme Court was about the scope and nature of the power of legislative privileges. While the State Legislative Assembly contended that this power was independent of the other provisions, the petitioners argued that the power, like all others in the constitution, was subject to the fundamental rights of citizens The court concluded that it had the power to review warrants issued by the Legislature for compliance with the due process requirements under Article 21, among others, thereby asserting the supremacy of the Constitution in general, and Art. 21 in particular, over the exercise of the privileges powers. The Supreme Court verdict in a Presidential Reference under Article 143 essentially held that the powers and privileges conferred on State legislatures by Article 194(3) were subject to the fundamental rights. The Supreme Court in another case held that Article 19(l)(a) would not apply and article 21 would. The Court further held: In dealing with the effect of Article 194 and the conflict pertaining to fundamental rights, an attempt will have to be made to resolve the said conflict by the adoption of the rule of harmonious construction - that is to reconcile the two. JMM case The case relates to certain MPs belonging to the Jharkhand Mukti Morcha and others allegedly taking bribes to vote for the Rao Government to save it from defeat in a no confidence motion in 1993. Supreme Court upheld Art. 105 which says that MPs can not be questioned for what they say or the manner of their vote in the Parliament by the judiciary. In the same judgement, Supreme Court had also ruled that MPs were public servants under the Prevention of Corruption Act. The Hindu case 2003

The Tamil Nadu Assembly passed resolutions in 2003 sentencing the publisher and four journalists of The Hindu to 15 days simple imprisonment for breach of privilege of the House after the Privileges Committee of the House pronounced them guilty. Codification of Privileges Constitution says (Art. 105) that powers, privileges and immunities of each House of Parliament and of the members and committees thereof shall be such as may from time to time be defined by Parliament. Constitution (Forty-fourth Amendment) Act, 1978 dropped the reference to the House of Commons that was originally there. The original reference was that where the privileges are not defined, they are similar to those of the House of Commons. However, no law defining the privileges has been made by Parliament so far. The Lok Sabha Privileges Committee headed by Kishore Chandra Deo (2008) in their report on Parliamentary Privileges Codification and related matters, said that there is no need to codify the privileges as the existing framework did well there had been only one case of admonition, two cases of reprimand and one case of expulsion for commission of breach of privilege and contempt of the House. Miscellany Whip The Concise Oxford Dictionary describes a Whip as an official appointed to maintain discipline among, secure attendance of, and give necessary information to members of his party He is central to the working of Parliament is the Whip. Though not officially recognised in the Rules of Procedure of the House, the efficient and smooth running of the parliamentary system depends largely upon the Whips. Each party has a whip or a number of whips depending on its numerical strength in the House Government. Chief Whip has the main function to ensure that the Government business is transacted in accordance with the planned programme. In managing the smooth passage of Government business, the Government Chief Whip has to ensure majority in every vote. He also has to ensure that there is always sufficient attendance of members to form a quorum and more particularly to give support to their own chosen speakers. In Indian Parliament the Minister of Parliamentary Affairs is the Chief Whip of Government. He is assisted by a few Ministers of State drawn from both the Houses. In the Rajya Sabha the Minister(s) of State in the Ministry of Parliamentary Affairs holds (hold) the position of the Government Whip. Under the Constitution (Fifty-second Amendment) Act, a member who votes or abstains from voting contrary to the whip (called Direction in the Act), runs the risk of losing his seat in the House. Thus, the written notice which a whip sends to member has assumed a constitutional status. Secretary General Next to the Chairman and the Deputy Chairman, the third important officer in the Lok Sabha / Rajya Sabha is the Secretary-General. He discharges all administrative and executive functions on behalf of and in the name of the Presiding Officer. The Secretary General is a permanent officer of the House and is appointed by the Presiding Officer. Parliament sits on an average 80-90 days in a year.

Private Members Bill Private Members are those members of the Parliament who are not ministers. They may also move a legislative proposal or Bill. In Lok Sabha, the last two and a half hours of a sitting on every Friday are generally allotted for transaction of Private Members Business, i.e., Private Members Bills and Private Members Resolutions. If there is no sitting of the House on a Friday, the Speaker may direct that two and a half hours on any other day in the week may be allotted for the transaction of Private Members Business. A member who wants to introduce a Bill has to give prior notice thereof. The period of noticer for introduction of a Bill is one month unless the Speaker allows introduction at a shorter notice. Presidents recommendation, if necessary, for introduction and/or consideration of the Bill should also be applied for by the member. Where a Bill, if enacted, is likely to involve expenditure from the Consolidated Fund of India, a financial memorandum giving an estimate of the expenditure involved has to he appended to the Bill by the member. In case the Bill contains proposals for delegated legislation, a memorandum regarding delegated legislation is also required to be appended to the Bill. The primary responsibility for drafting of Private Member& Bills is that of the members concerned The Lok Sabha Secretariat nevertheless renders necessary assistance in putting the Bill in proper form so that it is not rejected on technical grounds. A member cannot introduce more than four Bills during a session. When a Bill originating in the Lok Sabha is transmitted to the Rajya Sabha, in case of a Private Members Bill, a member of Rajya Sabba authorized by the Lok Sabha Member in charge of the Bill introduces the Bill. Bills seeking to amend the Constitution, apart from being subject to the normal rules applicable to Private Members Bills, have also to be examined by the Committee on Private Members Bills and Resolutions and only those Bills Which have been recommended by the Committee are put down in the List of Business for introduction

The Judiciary
Supreme Court The Supreme Court of India was constituted under Article 124 of the Constitution. It commenced its sittings on January 28, 1950. The original Constitution of 1950 provided for a Supreme Court with a Chief Justice and 7 puisne Judges -(puisne judge is a judge of the Supreme Court other than the Chief Justice). Parliament is given the power to increase this number there was progressive increase in the total strength of the apex court to 26 in 1986. Supreme Court (Number of Judges) Amendment Act, 2008 increased the number of judges to 31 including the Chief Justice of India. The proceedings of the Supreme Court are mainly in English and the procedure is regulated by the Supreme Court Rules, 1966. The Supreme Court of India comprises the Chief Justice and not more than 25 other Judges appointed by the President of India. Qualifications In order to be appointed as a Judge of the Supreme Court

a person must be a citizen of India and must have been: For at least five years, a Judge of a High Court or of two or more such Courts in succession, or An advocate of a High Court or of two or more such Courts in succession for at least 10 years, or in the opinion of the President, a distinguished jurist.

Appointment of Judges to the Supreme Court: Details Art. 124 of the Constitution of India deals with the appointment of Supreme Court Judges. Art 124(2) says that every Judge of the Supreme Court shall be appointed by the President by warrant under his hand and seal after consultation with such of the Judges of the Supreme Court and of the High Courts in the States as the President may deem necessary. In the case of appointment of a Judge other than the Chief Justice, the Chief Justice of India shall always be consulted. However, the actual process of appointment has gone through changes due to apex court verdicts.

Ques. 1 : What is Three Judge Case in the context of Indian judiciary and how it has influenced the appointment of judges of Supreme Court in India? Ans. In the SP Gupta case (1982) a seven-judge Constitution Bench held that the President is the final authority to appoint. He need not follow the advice of the judges whom he consults. In other words, consultation is not concurrence. It is known as the First Judges case. In 1993, a nine-Judge Constitution Bench of the Supreme Court in the Advocates on-record Association case, over-ruled the decision given in S.P. Gupta. The Supreme Court observed that when the President consulted the judges of the Supreme Court and the High Courts, the advice received by him is binding on him consultation is concurrence. It held that the recommendation for appointment should be made by the Chief Justice of India in consultation with his two senior-most colleagues and that such recommendation should be followed by the President. In case of any divergence between the judicial advice and the Presidential opinion, the former will prevail. Article 50 is quoted to give substance to the verdict divesting the executive of its judicial powers. It is known as the Second Judges Case. The 1993 decision was reaffirmed with minor modifications in 1998, on a reference made by the President under Article 143 of the Constitution. It was held that the recommendation for appointment etc should be made by the Chief Justice of India and his four senior-most colleagues (instead of the Chief Justice of India and his two senior-most colleagues) referred to as the Collegium for the purpose of appointment of Judges to the Supreme Court. It is known as the Third Judges Case. So far as the appointment of the Chief Justice of the Supreme Court of India is concerned, both the 1993 decision and the 1998 opinion lay down that the senior- most judge should always be appointed as the chief Justice of India. The Constitution also provides for the appointment of a Judge of a High Court as an ad-hoc Judge of the Supreme Court and for retired Judges of the Supreme Court or High Courts to sit and act as Judges of that Court. Retirement, resignation and removal Supreme Court judge retires when he attains the age of 65 years. He may resign addressing the letter to the President of India. He may be removed by an order of the President based on parliamentary vote. Removal of Supreme Court Judge Supreme Court Judge may be removed from his office by an order of the President passed after an address by each House of Parliament supported by a majority of the total membership of that House and by a majority of not less than two-thirds of the members of that House present and voting on the ground of proved misbehaviour or incapacity. Article 124(5) specifically Lays down that Parliament may by law regulate the procedure for the presentation of an address and for the investigation and proof of the misbehaviour or incapacity. In pursuance of Article 124

(5), Parliament passed the Judges (Inquiry) Act, 1968. The Judges (Inquiry) Rules, 1969 lay down the details of procedure for investigation and inquiry into the allegations against a judge. Judges (Inquiry) Act, 1968 regulates the procedure for the inquiry into an allegation of misbehaviour or incapacity of a Judge of the Supreme Court or High Court and for the presentation of an address by Parliament to the President for removing him from office, if charges have been proved. The Act authorizes the constitution of a three-member committee once a motion for presenting an address to the President seeking the removal of a Judge is admitted in Parliament. The motion can be admitted only if 100 Lok Sabha or 50 Rajya Sabha members propose it. The committee includes the Chief Justice or one of the Judges of the Supreme Court, a Chief Justice of one of the High Courts, and one distinguished jurist. The committee, after giving reasonable opportunity to the Judge concerned to defend himself/herself, has to submit its report to the Parliament. If the committee finds the Judge guilty- fully or partly, then its report, along with the motion, has to be considered by Parliament. The motion should be adopted by each House of Parliament by a majority of that House and by a majority of not less than two-thirds of the members of that House present and voting. The address shall be presented to the President during the same session of Parliament for the removal of the judge. The guilty Judge is removed from office by Presidential Order. Acting CJI Art.126 says that when the office of the Chief Justice of India is vacant or when he is not in a position to perform his duties, they are performed by such other judge of the Supreme Court that the President may appoint. Adhoc and Acting Judges Art.127 says that if there is no quorum of the Supreme Court judges to hold or continue any session of the Court, the CJI, with the previous consent of the President and in consultation with the Chief Justice of the High Court concerned can request in writing a judge of the high Court who is qualified to be a judge of the Supreme Court, to function as ad hoc judge of the Supreme Court. While so attending as the judge of the Supreme Court he shall have all the jurisdiction, powers arid privileges, and shall discharge the duties, of a Judge of the Supreme Court. Art. 128 says that retired High Court and Supreme Court judges may be requested by the CJI, with prior consent of the President to sit and function as the judge of the Supreme Court. Every such person so requested shall, while so sitting and acting, be entitled to such allowances as the President may by order determine and have all the jurisdiction, powers and privileges of, but shall not otherwise be deemed to be, a Judge of that Court. His consent is necessary for attendance as the acting judge. Seat of Supreme Court Art.130 says that the Supreme Court shall sit in Delhi or in such other place or places, as the Chief Justice of India may, with the approval of the President, from time to time, appoint. Benches of Supreme Court To dispose of the cases before Supreme Court the matters are placed before various Benches of Supreme Court.

The Bail applications in appeals are heard by single-Judge (also known as Chamber Judge). Most of the matters are decided by Division Benches of the Supreme Court- two judges. If the two judges disagree- which is rare- the view of the senior judge prevails. Three-Judge Bench - Matters placed before three-Judge Bench are considered priority matters. Appellate jurisdiction of the Supreme Court is given in Art. 132, 133 and 134 for Constitutional, civil and criminal cases respectively. Constitutional cases Art 132 of the Constitution provides for an appeal to the Supreme Court from any judgment, decree or final order of a High Court, whether in civil, criminal or other proceedings, if the High Court certifies that the case involves a substantial question of law as to the interpretation to the constitution. A substantial question of law means a question on which two or more High Courts have differed. Civil Cases Appeals lie to the Supreme Court in civil matters (Art. 133) if the High Court concerned certifies (a) that the case involves a substantial question of Law of general importance, and (b) that, in the opinion of the High Court, the said question needs to be decided by the Supreme Court. The term general importance means that the case holds interest for a wider section of the society other than the litigants. Criminal Cases According to Art. 134 and Criminal Procedure Code provisions, an appeal lies to the Supreme Court if the High Court a. has on appeal reversed an order of acquittal of an accused person and sentenced him to death or to imprisonment for life or for a period of not less than10 year, or b. has withdrawn for trial before itself any case from any court subordinate to its authority and has in such trial convicted the accused and sentenced him to death or to imprisonment for life or for a period of not less than 10 years, or c. certifies that the case is a fit one for appeal to the Supreme Court. Earlier, life imprisonment meant an imprisonment of 14 to 20 years, but of late, the courts are interpreting it as imprisonment until the end of the natural life of the convict, unless remitted. Parliament is authorised to confer on the Supreme Court any further powers in criminal jurisdiction, under Art. 134. All cases involving Constitutional Inter-pretation and Presidential references are placed before a five-Judge Bench, popularly known as a Constitution Bench. The largest Bench so far was the 13-judge bench that delivered the Kesavananda Bharati case verdict in 1973. There has not been a 13 judge bench since then or till then. Another landmark verdict Golaknath case (1967) was determined by eleven - Judge Bench.

Ques. 2 : Explain the original jurisdiction of the Supreme Court. Is the writ jurisdiction of the Supreme Court original and exclusive? If not why? The Supreme Court has original, appellate and advisory jurisdiction. Original Jurisdiction Original jurisdiction means that a case originates in the court. It may be exclusive or otherwise. Under Art. 131, exclusive original jurisdiction of the Supreme Court is one where no other court in the country enjoys the same power. It extends to all federal disputes- any dispute between the Government of India and one or more States or between the Government of India and any State or States on one side and one or more States on the other or between two or more States. It also involves clarification as to whether-a certain item is in the residuary category or not. To explain, the Constitution distributes legislative powers to the Union parliament and State Legislature in the VII Schedule. Any item that comes up subsequently and is not covered by the three Lists. Union, State and Concurrent is in the residuary category and belongs to the Union Parliament. The writ jurisdiction of the Supreme Court is original but not exclusive as the power is also available to the High Courts (Art.226). Article 32 of the - Constitution gives an extensive original jurisdiction to the Supreme Court in regard to enforcement of Fundamental Rights. It is empowered to issue writs in the nature of habeas corpus, mandamus, prohibition, quo warranto and certiorari to enforce them. Appellate Jurisdiction SLP The Supreme Court has a very wide appellate jurisdiction over all Courts and Tribunals in India as it may, in its discretion, grant special leave to appeal under Art. 136 of the Constitution from any judgment, decree, determination, sentence or order interim or final-in any matter from any Court or Tribunal in the territory of India. However, SLP cannot be filed to challenge any judgement, determination, sentence or order passed by any court or tribunal constituted under any law relating the Armed Forces. Any one can file an SLP if the High Court refuses leave to appeal to the Supreme Court But such an appeal should be made within 60 days of refusal. Art. 138 Art.138 says that the powers of the Supreme Court can be enlarged by the Parliament. Power to transfer a case Article 139A(1) of the Constitution provides that where cases involving the same or substantially the same question of law are pending before the Supreme Court and one or more High Courts or before two or more High Courts and Supreme Court is satisfied, on its own motion, or on an application made by the Attorney General of India or by a party to any such case, that such questions are substantial questions of general importance, the Supreme Court may withdraw the case or cases and dispose of all cases itself. Article 139A(2) of the Constitution provides that the Supreme Court may in pursuit of justice,

transfer any case, appeal or other proceedings pending before any High Court to any other High Court. Code of Civil Procedure and Code of Criminal Procedure provide that Supreme Court may transfer any case from a High Court or other subordinate Court in one State to a High Court or other subordinate Court in any other State. Art. 140 It enables Parliament to confer ancillary powers on Supreme Court consistent with Constitution to make the Court more effectively discharge its Constitutional duties. Art. 141 It says that the law declared by Supreme Court is binding on all courts within the territory of India. Art. 142 It says that the Supreme Court, in the exercise of its jurisdiction, may pass such decree or make such order as is necessary for doing complete justice in any cause pending before it, and it shall be enforceable throughout the territory of India. Advisory jurisdiction: Art.143 The Supreme Court has special advisory jurisdiction in matters which may specifically be referred to it by the President of India under Article 143 of the Constitution. According to Art. 143, if it appears to the President that

a question of law or fact has arisen, or is likely to arise which is of public importance and that it is necessary to take the opinion and advice of the Supreme Court on it

The President of India may seek the advice of the Supreme Court on such a matter. The Supreme Court may render its advice or it may decline to do so. But in one case such advice is to be mandatorily given pre-Independence agreements and accords that India entered into. The advice of the Supreme Court is not binding on the President but it has the value of a judgement. The advisory powers of the Supreme Court have been of enormous value so far in clarifying on various matters Constitutional and legal matters as can be seen from the list of references given below. Presidential references so far 2004 SYL canal and unilateral termination of all agreements by Punjab 2002 Gujarat Assembly and Art. 174 1998 Judges appointment 1993 Rama Janma Bhoomi

1991 Interim Order of Cauvery Water Disputes Tribunal 1990 Member, Punjab State Public Service Commission. 1983 Chairperson Punjab Public Service Commission 1978 The Special Courts Bill, 1978 1974 Presidential reference whether election to the post of President could be held when the Gujarat Assembly was dissolved. 1964 Parliamentary privileges and fundamental rights 1963 The Bill To Amend The Sea Customs act 1878 and Central Excises and Salt Act, 1944 1959 The Berubari Union and exchange of Enclaves 1958 Kerala Education Bill, 1957 1951 The Delhi Laws Act, 1912, the Ajmer-Merwara (Extension of Laws) Act, 1947 and the Part C States (Laws) Act, 1950 Art.144 Art 144 says that it is the duty of every person and authority in the country to act in aid of and render necessary assistance for the enforcement of the orders of the Supreme Court. Miscellaneous Powers Election disputes Article 71 of the Constitution, provides that all doubts and disputes relating to election of a President or Vice-resident are required to be enquired into and decided by the Supreme Court. Public Service Commission Members Removal Article 317 of the Constitution, provides that the Chairman or any other member of a Public Service Commission can be removed from his office by order of the President, on the ground of misbehavior, after the Supreme Court on reference being made by the President, has on enquiry reported that he ought, on such ground, to be removed from his office. Ques. 3 : Bring out the instances where the Supreme Court Judgement have became laws. Does this means that Supreme Court is an important arm of law-making? Ans. A judgement of the Supreme Court has the force of law. For example, the doctrine of basic features was introduced by the apex court in the Kesavananda Bharati case verdict in 1973. It is accepted as a part of law. Similarly, any law made by the legislature/ parliament can be added to or subtracted from, by the apex court. For example, the amendment act 2005, the supreme court made it operational subject to the concept of creamy layer.

In the ADR case 2002, it made it mandatory for a contestant in the Lok Sabha and Assembly elections to disclose certain details related to education, properly and criminal record. There was no law on the subject and the apex court filled the void. Similarly, it affirmed the power of the judiciary over cases of is qualification related to defections by setting aside para 7 of anti defection law in the Kihoto Hollohan case in 1 992.The apex court ruled that the power to disqualify on grounds of defection can not be taken away from the judiciary and the verdict of the Presiding Officer is subject to judicial review. Ques. 4 : Does the increasing instances of criticism of judiciary forms a contempt of court or is a part of popular will which is required to strengthen the parliamentary democracy in India? Ans. Contempt of Court Acts which willfully seek to disrupt the normal judicial process constitute contempt of court. In India, Supreme Court and High Courts are given the power to punish contempt of court as shown below: Article 129: The Supreme Court shall be a Court of record and shall have all the powers of such a court including the power to punish for contempt of itself. Article 215: Every High Court shall be a court of record and shall have all the powers of such a court including the power to punish for contempt of itself. Need for contempt of court powers Effective discharge of Constitutional and other legal duties demands that the majesty of law and the dignity and authority of the courts should be respected and protected. It in turn helps uphold the rule of law and Constitutional governance. Contempt powers are necessary for the higher courts to enforce rule of law and judicial orders contempt powers are a balance between the right of an individual to criticize the judiciary and the need of the judiciary to enforce respect for the law. Civil and criminal contempt Contempt of court may be civil contempt or criminal contempt Civil contempt means willful disobedience to any judgement, direction, order, writ etc of a court or willful breach of an undertaking given to a court. Civil Contempt normally attracts a fine and not imprisonment unless there are special circumstances. Criminal contempt means saying or doing anything that

scandalizes or lowers the authority of court, or interferes with the due course of any judicial proceeding, or obstructs the administration of justice in any other manner.

Willfulness is necessary to constitute contempt. Mere disobedience without a willful element is not sufficient to constitute contempt. Partial non-compliance of a court order also amounts to contempt.

Superior courts of record. -Supreme Court and High Court: set their own jurisdiction in the exercise of constitutional powers: which includes contempt powers. Their powers can not be limited by statute. Supreme Court has inherent power under Article- 129 of the constitution to take suo motu action to proceed against contempt.

Of itself Of High Court and Of a subordinate court. of itself as well as of subordinate courts.

High Court being a court of record has inherent power in respect of contempt

High Courts have power to punish for contempt of subordinate courts under Article-215 but that does not affect or abridge the inherent power of Supreme Court under Article-129. The Supreme Court and High Court both exercise concurrent jurisdiction under the constitutional scheme of punishing for contempt of any subordinate court and contempt of High Court. Generally, cases relating to contempt of subordinate courts are matters for High Courts. However, under rare circumstances affecting the entire judiciary, Supreme Court may directly take cognizance of contempt of subordinate courts. Defence Contemnor may defend himself on the basis of truth and public interest. Under the contempt of law, to protect principles of natural justice, it is necessary that the procedure is fair that the contemnor (one who commits contempt of court) is made aware of the charge against him and is given a fair and reasonable opportunity to defend himself. When a person defies the orders of a High Court in a place that is outside the Courts normal jurisdiction, the High Courts powers extend beyond the normal territorial limits to punish for contempt. Court of record Under Art. 129 of the Constitution the Supreme Court is a court of record. It means it has the following attributes

its proceedings are recorded and can be quoted as evidence in any court in the country. it sets its own jurisdiction and it can punish for contempt of court including contempt of itself.

Ques. 5 : An independent judiciary is essential for the strength of a federal democracy like India. Examine? Ans. An independent judiciary is essential for the strength of a federal democracy like ours. Our Constitution establishes it on the basis of the following

Appointment of judges of Supreme Court is kept above politics as the President appoints Supreme Court and High Court judges in consultation with the CJI and such other judges of the SC and HCs as he deems necessary Removal is possible on grounds of proved misbehaviour or incapacity and the parliament should vote with special majority followed by Presidential Order of removal Salaries etc are charged on the Consolidated Fund of India and are non-votable Administrative expenses of the Supreme Court are charged on the Consolidated Fund of India Conditions of service can not be varied to the disadvantage of judges after their appointment After retirement, Supreme Court judge can not practise in any court in the country and a High Court judge can not practise in the High Court where he retires Parliament can only enhance the powers of the Supreme Court and can not reduce the same Art. 141 says that the SC judgements are binding on all courts in the country

Ques. 6 : Critically examine the need for constituting All-India Judicial Service (AIJS) for improving the efficiency of the subordinate judiciary? Ans. The subordinate courts/subordinate judiciary is a State subject (an item in State List (List II). The appointment of the members of the subordinate judiciary is made by the Governor. Such appointment is to be made in the case of district judge, in consultation with the High Court and in the case of other posts, in consultation with the Public Service Commission and the High Court There has been a suggestion for many decades that an AIJS should be constituted to attract the best young talent that can improve the efficiency of the subordinate judiciary. Law commission recommended that AIJS should be constituted, essentially for manning the higher services in the subordinate judiciary. The Supreme Court in 1995 had endorsed the recommendation. Article 312 deals with the All-India Services. Constitution (Forty-second Amendment) Act, 1976 inserted All-India Judicial Service into the Article. The Amendment Act says that the All-India Judicial Service shall not include any post inferior to that of a district judge as defined in article 236. The process of creation of an All India Service needs to be noted. If the Council of States (Rajya Sabha) declares by resolution supported by not less than two- third of members present and voting that it is necessary or expedient in the national interest to do so, Parliament may by law provide for creation of an All India Judicial Service (AIJS) common to the Union and the States and also to regulate the recruitment and conditions of service of persons appointed to such All India service (Art. 312). Law of the Parliament to create AIJS is not to be deemed to be an amendment of the Constitution within the meaning of Article 368. Three main objections are raised in this matter

inadequate knowledge of regional language would dampen judicial efficiency promotional opportunities of the members of the State judiciary would be severely hurt

erosion of control of the High Court over subordinate judiciary would impair independence of the judiciary.

The objections have been answered effectively by, among others, the Law Commission. Learning the regional language has never been a problem as seen in our experience with the IAS and IPS. It also furthers cultural integration. With respect to the second objection, only a portion of the total vacancies are to be filled through the All India competitive examination while the remaining are open to be filled by promotion from the lower cadres. Control of the High Court will remain as, on allotment to a State, the allottees (members of AIJS) would become members of the State Judicial Service for all practical purposes. Ques. 7 : To ensure accountability and transparancy in the appointment and functioning of judiciary, it is necessary that it is made broad-based. Examine? Ans. The process of appointment to the higher judiciary and transfer of High Court judges has been a point of national discussion in the last many years, particularly the Second Judges case in 1993 when the primacy was given to the Collegium of judges in this matter. There is a large section of opinion that believes that members of the executive and the judiciary at the highest level should be involved in the process of appointment and transfer, for the process to be transparent and broad-based. Another issue related to higher judiciary is to ensure its accountability in matters of conduct. To address the twin issues, National Judicial Commission has been under consideration. The 67th Constitutional (Amendment) Bill, 1990 proposed the creation a National Judicial Commission composed of serving judges headed by the CJI for judicial appointments. But it lapsed with the dissolution of Lok Sabha. In 2003, the Constitution (98th Amendment) Bill was introduced in the Lok Sabha during the Budget session seeking to set up a National Judicial Commission (NJC) by including Chapter IV A in Part V of the Constitution with the following main functions

Appointment of Judges to the higher judiciary transferring High Court Judges draw up a code of ethics for Judges inquire into cases of minor misconduct of a Judge (major cases of misconduct attract removal under Art. 124) and advise the Chief Justice of India (CJI) or the Chief Justice of a High Court appropriately after such inquiry. CJI, who is chairperson two Judges of the Supreme Court next to the CJI in seniority Union Minister for Law and Justice and one eminent citizen to be nominated by the President in consultation with the Prime Minister, who will hold office for a period of three years.

The composition of the proposed NJC is


in the case of appointment or transfer of a High Court Judge, the Chief Justice of that court and the Chief Minister of that State (or the Governor, if - the State is under Presidents Rule) shall be associated with the NJC.

National Commission to Review the Working of the Constitution (NCRWC) (2002) recommended it. The Bill lapsed with the dissolution of the 13th Lok Sabha. Ques. 8 : The removal of judges has been one of the most cumbersome, there is a need to make it simpler and faster. Critically examine? Ans. Judges (Inquiry) Bill, 2006 : The Judges (inquiry) Bill, 2006 establishes a National Judicial Council (NJC) to conduct inquiries into allegations of incapacity or misbehaviour by High Court and Supreme Court judges. The NJC shall consist of the Chief Justice of India, two Supreme Court judges and two High Court Chief Justices to investigate High Court judges; or the Chief Justice of India and four Supreme Court judges to investigate Supreme Court judges. The NJC shall investigate complaints submitted by any person, or upon receiving a reference from Parliament based on a motion moved by 50 Rajya Sabha or 100 Lok Sabha MPs. It may also entertain complaints from any other source. The complaint has to be filed within two years of the alleged misdemeanor. If the complaint is found to be frivolous, the complainant may be punished with up to one year imprisonment and a fine up to Rs 25,000. After inquiry, if the Council is of the opinion that the charges proved do not warrant removal of the Judge, it may impose following minor measures:

issuing warnings; withdrawal of judicial work for a limited; request that the Judge may voluntarily retire; censure or admonition, public or private.

If the charges that are proved warrant removal in the opinion of the Council, Council shall advise the President accordingly. The President shall cause the findings of the Council to be laid before both Houses of Parliament. If both the Houses of parliament pass the resolution, the President removes the judge. There is judicial review after the minor punishment or removal. NJC is necessary to bring about accountability among the higher judiciary for actions which may not be grave enough to warrant impeachment but serious enough for minor punishment. Another purpose NJC serves is that it can also be involved in the impeachment process if the Parliament so desires. Criticism

It is considered inappropriate that NJC is entirely composed of judges. There should be wider participation in the process. Judicial review after Government action undermines the effectiveness of the Presidential Order.

Unlike the NJ Commission for which a CAB was introduced in 2003, the NJ Council has no role in the appointment of judges. The Bill lapsed with the dissolution of the 14th Lok Sabha.

Ques. 9 : 'Justice delayed is justice denied. In this context examine the causes responsible for pendency of cases and suggest resources to improve it? Ans. Supreme Court ruled that speedy trial is a part of right to life (Art. 21). Judicial delays and huge backlog have the effect of emboldening anti-social elements common man loses faith in the judicial system; loss of important evidence is possible because of fading of memory or death of witnesses. By 2009, over 25 million cases were pending in trial courts, while the number of cases pending in 21 High Courts stood at 3.7 million. Backlog is increasing for many reasons like

Increasing number of laws and increasing levels of literacy lead to mounting litigation judicial impact assessment(JIA) is inadequate. That is, it is not estimated for every Bill passed as to the impact on caseload, judges/staff, infrastructure and financial needs. Judge - population ratio is low at 13 judges per 1 million. Law Commission recommended to raise it 5-fold Approved strength of High Courts is 877 (2008) but there are about 300 vacancies. Similarly, against a sanctioned strength of 15,917 trial court judges, there are only 12,524 judges available leaving a shortfall of 3,393 judges (2008) Judicial infrastructure is inadequate - both in terms of courts or adoption of new technology (IT etc.) Procedures permit unwarranted adjournments (postponements)

Chief Justice K G Balakrishnan suggested higher budgetary allocation to set up new courts like evening courts and special magistrate courts to combat pendancy of cases. 59 lakh petty cases are pending and they can be disposed off in a short time, if special magistrates could be appointed. In Andhra Pradesh, morning courts are functioning before the normal office hours. In Gujarat, evening courts are functioning. They have disposed off lakhs of cases. Gram Nyayalayas Bill, 2007 aims to set up over 5000 courts to provide justice in relatively simple civil and criminal cases in 90 days. Pendency can also be reduced through alternative settlement of disputes mediation and conciliation. Lok Adalats have proved to be useful in mass disposal of cases. State governments should set up at least one family court in every district as it means adoption of a conciliatory approach in such cases, giving preference to mutual settlement over adjudication by court. The country now has only 190 family courts. The judiciary has recently adopted some measures, including increase of the working hours of High Court judges.

During the Tenth Plan (2002-2007) 0.078 per cent of the total plan outlay was spent on judiciary. It needs to be increased substantially for the reforms to take effect. Video-conferencing should be allowed in the judicial process. It is common for the criminal cases getting adjourned on account of inability of the police or jail authorities to produce the accused in court. Sometimes the witnesses are residing at far off places or even abroad. It is not convenient for them to attend the court. Video conferencing is a convenient, secure and less expensive option and can speed up the trial, among other advantages. National Judicial Infrastructure Plan prepared by the National Judicial Academy, Bhopal for upgrading judicial infrastructure to enable access to justice for common man is under consideration. The plan proposes new initiatives such as fast track courts, second shift in existing courts, etc for speedy disposal of cases Moily Commission Second Administrative Reforms Commission (ARC) headed by Veerappa Moily made the following recommendations

fixing a time limit for various stages of trial by suitable amendments to the Criminal Procedure Code cases under the Prevention of Corruption Act are held on a day-to-day basis for quick disposal guidelines to avoid unwarranted adjournments endorsement of Malimath Committee proposal for increasing the working days of High Courts.

Judicial Reforms In order to make the judiciary more responsive to the needs of people; speed up the process of justice; be accessible, accountable; improve the very low rate of prosecution in criminal cases; and reduce costs, the following must be done.

pendancy must be drastically pruned Computerization of the courts fill up vacancies in High Courts and subordinate courts. competent and able members of bar are to be attracted to the judicial posts the high level of court fees prescribed by many state governments must be reduced as it adds to the cost of justice judicial accountability needs to be strengthened witness protection Laws need to be modernized as some of them are more than 100 years old

Many reforms have already been initiated and are in progress. Some have already taken effect as shown below:

CPC amendments

CrPC amendments 2008 Lok Adalats Gram Nyayalayas Act 2008 B-judiciary It is suggested that the jury system like in the US should be adopted where the common public can be asked to work as jurors (judges) and decide cases on the basis of facts. It will reduce pressure on courts.

CPC Amendments 2002 In a move to speed up justice delivery, the Centre amended the Civil Procedure Code which provide for time-bound disposal of civil cases. The following amendments make for speedier disposal of cases

Only three adjournments are permitted. The court is also empowered to fix a time limit for oral arguments and to avoid delay, it may ask the parties to file written submissions. a judgment is to be pronounce within 60 days from the date on which the hearing concludes . ADR- conciliation and arbitration- should be encouraged It introduced plea bargaining to reform the criminal justice system prohibits arrest of a woman after sunset and before sunrise except under rare circumstances requires police to give information about the arrest of a person as well as the place where he is being held to anyone nominated by him The State government may establish a Directorate of Prosecution. allows the use of DNA and other techniques during medical examination of the accused. It entitles an under trial, other than those accused of an offence for which death penalty is prescribed, to be released with or without surety if he/she has been under detention for more than half the prescribed period of imprisonment. It also provides for release of under trials who are detained beyond the maximum period of imprisonment provided for the alleged offence.

Code of Criminal Procedure (Amendment) Act, 2005


The amended Act would tone up the investigating machinery and process, strengthen the prosecution machinery, tackle problems of under trials, safeguard the interests of women and stipulate bail conditions. Plea Bargain It is was introduced in India by amendment of the Code of Criminal Procedure. Under plea bargain, criminal defendant and prosecutor reach an agreement subject to court approval. The accused admits guilt without a trail, and return is given a lighter punishment.

This allowed for cases in which the maximum punishment is imprisonment for seven years. However, offences in socio-economic area like sati are not covered and offences committed against a woman or a child below the age of fourteen are also excluded. The rules say the court would examine the accused in camera to determine if he has willingly opted for plea bargaining. Critics of the system point out that it puts strong pressure on defendants to plead guilty to crimes that they know that they did not commit. Furthermore, the system encourages lawyers to overcharge. Ques. 10 : The Criminal Procedure Code (Amendment) Act, 2008 provides for a number of path-breathing measures in the existing law. Discuss? Ans. Major changes in the Criminal Procedure Code were made to take away the powers of the police to arrest in cases of alleged offenses which carry a maximum sentence upto seven years of imprisonment. Once the law, CrPC (Amendment) Act 2008, becomes effective, the police, instead of arresting the accused, will be obliged to issue him/her a notice of appearance for any offence punishable with imprisonment up to seven years. The person can be arrested only if he/she does not appear before the police in response to the notice. Seven years or less is the maximum penalty applies for many offences. These offences include such as attempt to commit culpable homicide, kidnapping, death by negligence, cheating, voluntarily causing grievous hurt, outraging a womans modesty, robbery, attempt to suicide. These amendments have been made in section 41 of the CrPC. Under Section 41, as it originally stood, a police officer may, without an order of magistrate and without a warrant, arrest any person concerned in any cognisable offense. The rationale of the amendment mentioned abovein section 41 of the code of criminal- procedure is its misuse. The amendment in CrPC, however, allows police to arrest without an order from a magistrate a person who commits a cognisable offense in the presence of a police officer. It, however, allows arrest of a person who has committed a cognisable offence- punishable for a term which may be less than 7 years or extend upto 7 years, there are reasonable grounds to suspect such arrest is necessary for proper investigation of the offence or for preventing tampering with the evidence. The only additional requirement in such cases is that the police officer will have to record his reasons for making the arrest. The amendments in the Code of Criminal Procedure Code (CrPC) also provide for a number of path-breaking measures in the existing law to lessen the-agony of rape victims. Completion of trial of rape cases within a period of two months from the date of commencement of the examination of witnesses, hearing of the cases by woman judges (as far as practicable) and questioning of rape victims in the presence of her parents or a social worker are some of the provisions of the CrPC (Amendment) Act 2008. It also has provisions of commuting of capital punishment of a pregnant convict to life imprisonment rape victims rights to appeal against acquittal, beginning of a. rape victim compensation scheme, audio-video recording of statement of witnesses and trial of cases

through video-conferencing in rape cases. As far as bringing changes in the law concerning rape victims is concerned, the Act also provides for investigation of a case at the residence of the victim and as far as practicable by a woman police officer. These provisions will spare the victim of the agony of visiting the police station and answering inconvenient questions in front of unidentified persons. The Supreme Court of India has voiced in its various landmark judgments that, Arrest of a person is a very serious matter and a wrongful arrest causes grave ignominy to the person. The third report of the National Police Commission, referring to the quality of arrest by the police in India had mentioned that power of arrest was one of the chief sources of corruption in the police. The report suggested that by and large nearly 60% of the arrests were either unnecessary or unjustified and that such unjustified police action accounted for 43.2% of the expenditure of the prison department. The Act mandates the State government to establish police control rooms at the district and State levels and display on notice boards kept outside the control rooms the names and addresses of the persons arrested, and the names and designations of the police officers who made the arrests. The law provides for payment of compensation to victims for illegal arrest and police harassment. Every State in coordination with the Centre should prepare a scheme for providing funds for compensation to the victim or his/her dependants who suffered loss or injury as a result of the crime and who require rehabilitation. Lawyers opposed the amendment to Section 41 of CrPC as it is totally not in the interest of curbing the crime itself, according to K. Pardhasaradhi, secretary. They claimed that the wealthy would make use of the lacuna in the section to escape punishment and that the poor only would be arrested in the name of notice of appearance before the police officer. Ques. 11 : Inexpensive justice to people is the essential for making justice accesible to people. In this context eleborate on the measures taken by the government? Ans. Gram Nyayalaya Act 2008 aims at providing inexpensive justice to people in rural areas on their doorstep. It provides for first class judicial magistrates dispensing justice. At least 3000 judges, who will be the judicial magistrate first class (JMFC) will be appointed; They will be called Nyaya Adhikaris Nyaya Adhikaris will be appointed by the states in consultation with the high courts. They are strictly judicial officers. They will be drawing the same salary, deriving the same powers as the first class magistrates working under the High Courts. Gram Nyayalayas will try criminal cases, civil suits, claims or disputes concerning all the offences not punishable with death, imprisonment for life or imprisonment for a term exceeding two years. Theft, receiving or retaining stolen property, assisting in the concealment or disposal of stolen property where the value of property does not exceed Rs 20,000 -, dispute relating to purchase of

property, cultivation of land, right to draw water from a tubewell or well are some of the offences which could be tried in the Nyayalayas. An appeal from the judgment of the Gram Nyaylaya will lie with the sessions court which will be heard and disposed of within six months from the date of filing of the appeal. For the Grain Nyayalayas, the Centre will bear the full cost on capital account. These courts will sit at the district headquarters and in taluks. They will go in a bus or jeep to the village, work there and dispose of the cases. The cost of litigation would be borne by the state and not by the litigant. Rs. 6.4 lakh would be the recurring expenditure, including salary of the staff, per annum while total capital expenditure would be met by the Centre. Tribal areas are not covered Fast Track Courts Fast track courts were set up, on the recommendation of the 11th Finance Commission to deal with criminal cases involving undertrials (there are 1.8 lakh undertrials in jails around the country) and other cases pending for more than two years. The aim is to setup five fast track courts in each district.As per the latest available information received from the High Courts/State Governments, 32.34 lakh cases have been disposed off by these courts, out of 38.90 lakh transferred to these courts leaving 6.56 lakh cases pending for disposal. The scheme of central assistance for Fast Track Courts was extended for a period of one year i.e. upto 31.3.2011. It was decided that there will be no central funding for Fast Track Courts beyond 31-03-2011. Alternative Dispute Resolution Alternative dispute resolution encompasses a range of means to resolve conflicts short of formal litigation. The modern ADR movement seeks to reduce cost and delay and avoid adversarial nature of litigation. The interest in ADR essentially centres around Lok Adalats. ADR today falls into two broad categories

court-driven options and community-based dispute resolution mechanisms (Lok Adalats) Court-driven ADR includes mediation/conciliationthe classic method where a neutral third party assists disputants in reaching a mutually acceptable solution.

Supporters argue that such methods decrease the cost and time of litigation, improving access to justice and reducing pressure on courts, while at the same time preserving important social relationships for disputants. Community-based ADR is often designed to be independent of a formal court system that may be expensive and inaccessible. India set up lok adalats in the 1980s.

Arbitration, conciliation and negotiation: Conciliation Conciliation is an informal process designed to create an environment where negotiations can take place. If the parties fail to reach and agreement, the case is referred to mediation. Mediation Mediation is a voluntary and confidential process where a neutral third party assists negotiation. The parties are responsible for reaching an agreement and the mediator cannot impose a settlement. The mediators role is to facilitate communication, promote understanding, and use problems solving techniques with the goal of assisting the parties to reach their own agreement. If the mediation fails to reach agreement, the case is referred to arbitration. Arbitration Arbitration is a form of private adjudication where a mutually acceptable third party hears arguments from either side in a dispute, and renders a judgment. The judgment, known as an award, is confidential and binding. Ques. 12 : What are Lok Adalats and how these adalats have helped in improving justice to the people? Ans. Lok Adalat literally means "people's court. It is an alternative dispute settlement mechanism which settles disputes through conciliation and mediation. It helps in quick disposal of cases and the process is simple and carries no fees. Lok Adalats are statutory forums since the enactment of Legal Services Authorities Act, 1987. All legal disputes pending in civil, criminal, revenue courts or a tribunal can be taken to Lok Adalat for amicable settlement except criminal cases which are non-compoundable (that is, serious offences where charges cannot be dropped without the consent of the judge). Legal disputes can be taken up and settled by Lok adalats at pre-litigative stage also i.e. before the parties have entered into litigation by filling a case in a regular court. Lok Adalats, generally, consist of a judicial member a legal practitioner and a social worker (generally, a woman). They follow their own procedure. They have the power of a Civil Court, in respect of summoning of evidence and, examination of witnesses, requisitioning of public records, etc. No lawyers are involved in the process. The procedure, followed in the Lok Adalats for the settlement of cases, is simple, informal and flexible. In 2008, the Supreme Court ruled that the Lok Adalats set up under the Legal Services Authority Act have no adjudicatory or judicial functions and they cannot decide cases referred to them on merits. A Bench comprising Chief Justice K.G. Balakrishnan and Justices G.P. Mathur and R.V. Raveendran said that "Lok Adalat determines a reference on the basis of a Compromise or settlement between the parties at its instance and puts its seal of confirmation by making an award in terms of the compromise or settlement.

But when the Lok Adalat is not able to arrive at a settlement, the case would be returned to the court from which the reference was received for disposal in accordance with the law. No Lok Adalat has the power to hear parties to adjudicate cases as a court does. it discusses the subject matter with the parties and persuades them to arrive at a just settlement. If the case is settled in the Lok Adalat, that is, if the litigants agree to a settlement in the Lok Adalat, it will have to be complied with, within a month. It is enforceable like the decrees of a civil court and are binding on the parties to the dispute. It is final as there does not lie any appeal against it. Lok Adalats are monitored by the State Legal Aid and Advisory Boards. Parliament in 2002 made the Legal Services Authorities (Amendment) Act 2002, which provides for the constitution of permanent Lok Adalats. The Act seeks to establish permanent Lok Adalats with maximum monetary jurisdiction of Rs. 10 Iakhs for conciliation and settlement of cases relating to public utility services like electricity boards, transport corporations etc. In their conciliatory role, the Lok Adalats are guided by the principles of justice, equity and fair play. The Lok Adalats have delivered inexpensive and expeditious justice and need to be extended further. Judicial Accountability In a Constitutional democracy, every institution is subject to accountability, including the judiciary. Accountability of the judiciary in respect of its judicial functions and orders is provided for by an appeal and review of orders. The mechanism for accountability for serious judicial misconduct, for disciplining errant judges is removal, in the Indian Constitution (Art.124). It is so difficult that it is not practical and so does not function as a deterrent. Therefore, National Judicial Council is being considered for minor offences. The Judges Inquiry Bill 2006 introduced in the Parliament is an important step in this direction as it seeks to set up NJC. The National Commission to Review the Working of the Constitution (NCRWC) made similar recommendations in the matter in its report in 2002. The accountability question has the following dimensions: Judicial accountability under which the lower court verdicts are open to challenge and nullification by the higher court. In matters related to death sentence and other punishments handed by the judiciary, there is Presidential mercy available under Art.72. Where the judicial members suffer from misbehavior or incapacity, Art. 124 prescribes removal by Parliamentary address and Presidential order. Ques. 13 : Judicial activism as a corollary of Public Interest Litigation has on the one hand ensured justice to the weak and vulnerable and on the other has checked the executive from being arbitrary. Critically examine the statement in context of the instances of judicial overreach. Ans. Judicial review is the power of the judiciary to review the laws made and executed by the legislature and executive to make sure that they are in line with the Constitution and statute. If

they are not, judiciary strikes them down partly or wholly. The power of judicial review is given to the judiciary by various provisions of the Constitution and law. For example, Art. 13 says that no law is valid if violates Fundamental Rights. Art.131 says that if there is a federal dispute between states and centre or between states, Supreme Court has exclusive power to settle it. Art.32 and 226 give power to the Supreme Court and High Courts to restore Fundamental Rights (Supreme Court) and all rights (High Courts) in case they are violated. The PIL movement in the country is a classical case of judicial activism whereby the judiciarythe higher rungs- take justice to the doorstep of the weak and vulnerable. Since the exploited and the illiterate do not have the means of moving the courts for their rights, the apex court allowed any public spirited body to take up the case on their behalf. It is a case of the judiciary being actively interested in taking justice to the door step of the marginalized. The Supreme Court since late seventies has been expanding the scope of FRs - particularly Art. 21 (right to life and personal liberty). It is also interpreted as judicial activism. Judicial overreach, on the other hand, is a case of judiciary encroaching into the territory of the other two organs- legislature and executive. Continuous mandamus, questioning the expulsion powers of the legislature and so on are a part of it, according to the former Speaker of Lok Sabha, Shri Somnath Chatterjee. While activism is welcomed as it helps the weak, keeps the Executive on its toes and makes it efficient and by and large sustains the faith of the people in the Government, it is criticized for the following reasons.

Judiciary has no resources to monitor the tasks it assigns to the executive Judiciary can not think that it can solve all the problems It upsets the delicate balance among the three organs of the government Judiciary must turn its attention to solving its own problems like arrears.

Public Interest Litigation Justice Krishna Iyer, in Mumbai Kamgar Sabha vs. Abdulbhai Faizullabhai (1976) used the expression PIL for the first time. Justice Bhagavathi added momentum to PIL in the late seventies. PIL must be differentiated from private litigation. In private litigation, courts are approached for the redressal of wrong or injustice of a private person (or a company which is a legal person). He has to show that he has locus standi- that he is connected to the case and has the right to act or be heard. No one can approach the court without locus standi. But in PIL, the victims of violation of constitution and law may be weak, vulnerable and illiterate. There are many cases where public interest is violated-for example, child labour, bonder labour; criminals in election process; environmental damage, pollution, children not being able to go to school, people in high places being corrupt and so on. In such a case involving public interest, Supreme Court since late 1970s, allowed the principle of locus standi to be set aside. Any socially spirited individual is allowed to bring it to the notice of the court. Procedural rigidities have not been insisted on. The reason is that the victims are weak and illiterate and can not approach the courts themselves. The aim is to bring justice to the doorstep of the weak. It is called public interest litigation or social interest litigation.

PIL means a legal action initiated in a court of law for the enforcement of public interest in which the public as against private individuals have interest in the form of protection/restoration of their rights. It is meant to catalyse progressive socio-economic change; make administration responsive; lead to better environmental practices ; make civil society active; and so on. In the famous Asiad Labour case 1982, the apex court accepted a letter written by an NGO as writ petition and ruled in favor of workers. Newspaper reports can also be the basis for initiation of action. Affidavits have not been asked for. There are many cases in which the SC and the HCs ruled for popular welfare tightening norms for the blood banks; coming to the rescue of the under-trials who were imprisoned without trial for unduly long periods, setting guidelines for introduction of CNG based buses etc in Delhi; cleaning Yamuna etc; directing Delhi industrial units to lake care of the child labourers; release of bonded labourers; recently( 2008) the construction of Mundra SEZ was stopped in Gujarat by the Supreme Court on a PIL filed by fishermen; In 2009, The Supreme Court asked the Union government to respond to the PIL accusing it of inaction in bringing back black money kept secretly in foreign banks by Indians. Public interest Litigation is the power given to the public by courts through judicial activism. It is a case of judicial activism as the judiciary activates the public to approach the courts in social interest. While the PIL instrument has great potential to help the ordinary people, there is concern about misuse of PIL

it is being used for publicity; private interest is being projected as public interest individuals and organizations are trivializing and politicizing the PIL by questioning various government decisions without justification has led to loss of precious court time It has become a tool for obstruction, delay and sometimes, harassment.

The Supreme Court ruled that PIL was not a fundamental right implying that the courts had the right to reject appeals on the basis of public interest. A two judge Bench of the apex court (2008) observed that frivolous PIL cases should be imposed a penalty of Rs 1 lakh. There is a wing in the apex court that screens out frivolous PILs. Some frivolous PILs are: India should be renamed Hindustan; the Arabian Sea should be called Sindhu Sagar; the national anthem Jana Gana Mana should be replaced by the one offered by the petitioner (and partly sung before the Chief Justice); the Prime Minister should be summoned to the court to respond to these requests; and so on. Ques. 14 : Seperation of power is the basic feature of the Constitution which of only has been breached. Elaborate? Ans. Indian Constitution provides for a parliamentary democracy and the essential features of federalism. Separation of powers among the three organs of the government is a basic feature. There is a clear and delicate balance of power between the three organs. In their respective jurisdiction, the three organs are independent and the Constitution bars any interference.

Articles 121 and 211 bar the legislature from discussing the conduct of any judge in discharge of his duties except when impeachment proceedings are being take up, Articles 122 and 212 on the other hand preclude the courts from interfering in the internal proceedings of the legislature. Article 105 (2) and 194 (2) privileges protect the legislators from interference of the Courts with regards to freedom of speech and freedom to vote. However, in recent years, the phenomenon of judicial activism was seen to disturb the delicate balance and the judiciary overstepped into the domain of the legislature. For example, in 1998, when there was contention for the Chief Ministership of Uttar Pradesh Jagadambika Pal and Kalyan Singh, the apex court directed the Speaker to conduct the Composite Floor Test in the State Assembly. A similar situation in 2005 in the case of the Jharkhand Assembly, the apex court directed the Protem Speaker to conduct a Composite Floor Test to ascertain the majority in the House. The recent verdict of the Supreme Court on the expulsion of MPs from Parliament is another such instance There have been other incidents where courts and legislators found themselves on opposite sides. In 2003, on the issue of the mandatory disclosure of educational qualification, assets/liabilities and the details of criminal antecedents (if any) by candidates filing nominations for elections, there was a series of assertions and counter-actions by Parliament and the apex court, in which the judges finally prevailed and establishing the need for such disclosures. Similarly, the very recent verdict of the SC on extending the concept of a creamy layer to reservations for Scheduled Castes and Scheduled Tribes, and on invalidating the immunity of legislations (passed after 1973) placed in the Ninth Schedule of the Constitution from judicial review, were not taken well by Parliamentarians who treated these verdicts as usurpation of power by judges. Comments on judicial over-reach: In the Conference of Presiding Officers of Legislative Bodies convened by the Speaker of the Lok Sabha, Somnath Chatterjee in 2005, the Presiding Officers expressed concern over court orders that disturbed the delicate balance of power between the legislature and the judiciary. They drew attention to the gradual ascendancy of the judiciary over the other two other branches.

Former Attorney General of India Soli Sorabjee said judges must not instill in themselves that the judiciary can solve all problems. In the conference of Chief Ministers and Chief Justices held in New Delhi recently the Prime Minister cautioned the Courts not to cross their limits. His remarks came in the backdrop of various legislations being struck down by the Courts; the most recent being the stay on the implementation of 27 per cent reservation for Other Backward Castes in Centrally financed elite educational institutions. Protection of journalists who are ordered to be punished by the legislature and are protected by the courts as Art. 21 demands that procedure established by law is followed and there is no such statutory procedure laid down in the case of privileges as they are not codified Continuous mandamus is being followed occasionally by the courts.

Other instances of tension between the Two Institutions are

Continuous mandamus If any public or semi-public authority refuses to discharge its duty, high Court or Supreme Court can be approached to issue mandamus writ to enforce the duty. It is one time order. Not acting on the writ constitutes contempt of court and is punishable. Under continuing mandamus, Courts keep on passing orders and directions with a view to monitor the functioning of the executive. The executive becomes continuously accountable to the court. Recent instances have shown that, courts resorted to continuous mandamus in rare cases in public interest. But it OSCS problems like disturbance in the separation of powers; officers being accountable to judiciary instead of the political executive while the political executive is accountable to the legislature; power without responsibility; concentration of powers in one organ and so on. Ques. 15 : Judicial Impact Assessment (JIA) of every bill will go a long way in solving backlog of cases in Indian Courts? Ans. The Task Force constituted by Government to study the feasibility of Judicial Impact Assessment in India under chairmanship of Shri Justice M. Jagannadaha Rao has submitted its report in 2008. Mandatory judicial impact assessment of every bill will have a fair estimate of extra caseload, judges/staff, infrastructure and financial needs a new law is likely to generate. The committee recommended that Judicial Impact Assessments must be made on a scientific basis for the purpose of estimating the extra case-load which any new Bill or Legislation may add to the burden of the Courts and the expenditure required for adjudication of such cases must be estimated by the Government and adequate budgetary provision must be made therefore. Such impact assessments must be made in respect of Bills that are introduced in Parliament as well as Bills introduced in the State legislatures. If implemented, it will be for the first time that India follows a system that is in vogue in the USA. The panel recommended that the Centre must establish additional courts for implementation of central laws made in respect of subjects in the Union List. The expenditure on fresh cases likely to be added to the Supreme Court and high courts by new laws must be reflected in the Financial Memoranda attached to the Central State Bills. The panel proposed setting up of a judicial impact office in Delhi and similar offices in states to carry out the assessment by involving social scientists, legal experts and NGOs. Indian courts are disposing of 1.5 crore cases annually and there were 2.5 crore eases still pending in lower courts. The. backlog does not get wiped out because fresh cases...get filed in courts every year. There is no point in blaming the judiciary for case arrears, the blame must also lie with other departments that help (create) it, The report said. Ques. 16 : Malimath Committee on Criminal Justice System. Its recommendations? Ans. Criminal justice system involves the police, prosecution, judiciary and the jails along with the witnesses. If it functions well efficiently, crime rate will decline and conviction rate increases. Otherwise, the country faces the risk of erosion of peoples faith with the increase in the crime rate. The Malimath Committee, constituted in 2000 to recommend revamping of the criminal justice system in the country gave its report in 2003.The two volume report makes 158 recommendations with regard to the police, prosecution, the judiciary and criminal jurisprudence. It has, to a large extent, incorporated the recommendations made by in various reports of the Law Commission and the National Police Commission.

It recommended the following: Shift from the current adversarial system to inquisitorial system where the court be empowered to summon and examine as a witness any person it considers appropriate and to issue directions to the investigating officers as may be necessary to assist it in its search for the truth. The right of the accused to silence (the right not to be compelled to be a witness against oneself) must also be amended with the court given the right to draw adverse inferences if he refuses to answer the questions put to him by the court. The committee also concluded that the current standard of proof beyond reasonable doubt put a very unreasonable burden on the prosecution. It has suggested that the standard of proof be set midway between the current standard in India and the much lower standard current in continental Europe, namely preponderance of probabilities, at clear and convincing proof. With reference to the investigation of crimes, the committee called for a separation of the investigating wing of the police from the law and order wing. To ensure that the investigating agency was insulated from extraneous influences it has suggested the setting up a National Security Commission and State Security Commissions. It suggested setting up-permanent benches in the Supreme Court and High Courts to deal with criminal cases to be presided over by judges specialised in criminal jurisprudence. The committee recommended that evidence recorded in video and audiotapes before a police officer of the rank of a superintendent should be admitted as evidence. At present, confessions recorded by police are not admissible as evidence. He also recommended that from now on, criminal laws be reviewed every 15 years so that they are in tune with the changing times. It recommended amendments to Section 125 of the Criminal Procedure Code (CrPC) so that a woman living with a man like his wife for a reasonably long period is also entitled to the benefit of maintenance. It further suggested life sentence for the rapist and it should not be commuted; lenient law for women and child convicts. Other recommendations are: Special Courts Special courts are set up under various acts to expedite justice. The special courts are created under Scheduled Castes and Tribes (Prevention of Atrocities) Act, 1989. Immoral Traffic Prevention Act 1956 provides for special courts. The National Investigation Agency set up in 2008 after the Bombay terror attacks in November 2008 allow special courts to be set up and function continuously. Evening courts in Gujarat and Fast Track courts are also special courts. Special courts essentially speed up justice. Difference between a Criminal Case and Civil Case - There are two fundamentally different types of court cases - criminal and civil. Among the important differences between criminal and civil cases are these:

In a criminal case a State prosecutor, not the crime victim, initiates and controls the case. This method of beginning the case contrasts with civil cases where the injured party is the one who files the case. A person convicted of a crime may pay a fine or be incarcerated or both People who are held responsible in civil cases may have to pay money damages or give up property, but do not go to jail or prison.

Advocates on Record Only these Advocates are entitled to file any matter or document before the Supreme Court. They can also file an appearance or act for a party in the Supreme Court. Curative petition The apex court may entertain a curative petition and reconsider its judgment, in order to undo gross miscarriage of justice. Such a petition can be filed only if a Senior Advocate certifies that it meets the requirements of this case. Such a petition is to be first circulated before a Bench comprising of three senior most judges and such serving judges who were members of the Bench which passed the judgment/order, subject matter of the petition. Ques. 17 : Information technology is the hub around which spokes of judiciary will revolve in future? In this context elaborate on the steps taken in Indian judiciary in this direction? Ans. E-judiciary The e-Judiciary initiative is taken up- computerization and connectivity to help in meeting the needs of the citizens in a transparent manner and enable quicker disposal of cases. The Supreme Court took up the e-courts project under the-National e-Governance Plan (NeGEP) for linking about 15,000 courts in the country. It is planned to provide necessary infrastructure and concentrate on capacity building, judicial process from filing to execution and finally making the information available online between the courts, prosecuting and investigating agencies, persons, land records and registration offices thereby accelerating disposal of civil and criminal cases. From the time the case is filled till it is disposed of with judgment, the entire processing must take place electronically. This will enable easy search, retrieval, grouping, information processing, judicial record processing and disposal of the cases The proposed action for connecting all the 15,000 courts from the District Court to the Supreme Court through will enhance the efficiency of the judicial system. COURTIS (Court Information System) Project. Court is project undertaken by MC has streamlined registries at various courts. With the implementation of the system the number of pending cases in the Supreme Court has come down. COURTNIC COURTNIC is an information system designed to provide the information, on the status of cases in the Apex Court to a wide variety of users, from anywhere in the country. Computerisation of all 21 High Courts and 10 Benches on the lines of Apex Courts Computerisation has been done. All High Courts Cause List are also available on Internet.

District Courts Computerisation In 1997, NIC took tip the computerisation of all 430 District Courts in the country on the lines of High Courts Computerisation Project. CASE-STATUS This website provides Supreme Courts pending and disposed case status information to litigants/advocates on Internet. JUDIS NIC brought out Judgement Information System (JUDIS) on CD-Rom consisting of complete text of all reported judgement of Supreme Court of India from 1950 to 2000. The Judgements of 2001 onwards are available on Internet. Cause Lists on Internet Cause lists contain information on the scheduling of cases to be heard by the courts on the following day. Daily Orders on Internet The daily orders of Supreme Court, Delhi High Court are available on the web, immediately after they are signed by the Judges. High Court High Court stands at the head of the States judicial administration. There are 21 High Courts in the country. High courts have jurisdiction over a state, a union territory or a group of states and union territories. Three High Courts have jurisdiction over more than one state Bombay High Court has the jurisdiction over Maharashtra, Goa, Dadra and Nagar Haveli and Daman and Diu. Guwahati High Court, which was earlier known as Assam High Court, has the jurisdiction over Assam, Manipur, Meghalaya, Nagaland, Tripura, Mizoram and Arunachal Pradesh. Punjab and Haryana High Court has the jurisdiction over Punjab, Haryana and Chandigarh. Among the Union Territories, Delhi alone has a High Court of its own. Other six Union Territories come under jurisdiction of different state High Courts. Each High Court comprises a Chief Justice and such other Judges as the President may, from time to time, appoint. High Courts are sanctioned under Part VI, Chapter V, Article 214 of the Indian Constitution. The Chief Justice of a High Court is appointed by the President in consultation with the Chief Justice of India and the Governor of the state. Since 1993, after the Second Judges case, the system of collegial appointment began. Every High Court shall consist of a Chief Justice and such other judges as the President of India from time to time deem it necessary to appoint. High Courts are headed by a Chief Justice. The Chief Justices are ranked 14 (in their state) and 17 (outside their state) in the Indian order of precedence. The number of judges in a court is decided by dividing the average institution of main cases during the last five years by the national average.

The Additional Judges are appointed for a period not exceeding two years taking into account the temporary increase in the business of the High Court. Such judge shall also not hold office after attaining the age of 62. The Calcutta High Court is the oldest High Court in the country, established in 1862. Each High Court has powers of superintendence over all courts within its jurisdiction. High Court judges retire at the age of 62. To be eligible for appointment as a judge, one must be a citizen of India and should have held a judicial office for 10 years or must have practiced as an advocate of a High Court or two or more such courts in succession for a similar period. The High Court is a Constitutional Court in terms of Article 215. It is a court of record and has all the powers of such court including the power to punish for contempt of itself. The High Courts entertain all cases including the Constitutional cases except the federal ones which go exclusively to the Supreme Court. High Courts can try all offences including those punishable with death. High Courts, however, mainly deal with appeals from lower courts and writ petitions in terms of Article 226 of the Constitution of India. Each High Court has power to issue any person or authority and government within its jurisdiction, direction, orders or writs, including writs which are in the nature of habeas corpus, mandamus, prohibition, quo warranto and certiorari, for enforcement of Fundamental Rights and for any other purpose. The High Court shall also have superintendence over all Courts and Tribunals throughout the territory in relation to which it exercises jurisdiction, as provided in Article 227 of the Constitution. Tenure The Judges of High Court can remain in office till the completion of 62 gears of age. Besides on the following grounds he can be relieved from this office. 1. 2. If he is promoted and transferred to Supreme Court, If on the basis of misbehavior or disqualification, if the Parliament in a Single Session passes a resolution by a majority of the total membership which is more than two-thirds of the members present and voting in each House of Parliament separately. If the resign. After his retirement, he can practice in any High Court other than his own court. Salary etc.

3.

There is a provision for the payment of uniform salaries to the judges of all High courts in India. The Parliament approved a bill facilitating salary hike of the Supreme Court and the High Courts judges in 2009. The Act increased the salary of the Chief Justice of India (CJI) from Rs 33,000 to Rs. 1 lakh month, while other Apex court judges will get Rs. 90,000 from their earlier Rs 30,000 per month salary. It increased the salary of the Chief Justice of High Courts from Rs 30,000 to Rs 90,000 per mouth, and HC judges salary from Rs 26,000 to Rs 80,000 per month.

Below the High Court at the district level, the District and Sessions Judge heads the subordinate judiciary. He is known as a District Judge when he presides over a civil case, and a Sessions Judge when he presides over a criminal case. At the district and sub-district level, below the High Courts, stand the subordinate courts- the civil courts, family courts, criminal courts etc. Benches: permanent and circuit If a High court entertains a large number of cases of a particular region, it has a permanent bench (branch). Benches also exist in states which come under the jurisdiction of a court outside its territorial limits. Smaller states with few cases may have circuit benches. Circuit benches (circuit courts) hold proceedings for a few selected months in a year. Thus cases built up during this interim period are judged taken up when the circuit court is in session. In 2008, in Dharwad and Gulbarga in north Kamataka, circuit benches were set up. Article 214 of the Constitution of India says that there shall be a High Court for each State. Article 230 enables the Indian Parliament, by law to extend the jurisdiction of a High Court to any Union Territory. By virtue of this the High Court of Kerala is also the High Court having jurisdiction over the Union Territory of Lakshadweep. Appointment of Acting Chief Justice Under Article 223, if the office of the Chief Justice of any State High Courts falls vacant or if the Chief Justice is absent or for some reason is not in a position to discharge the responsibilities of the office, then the President is empowered to appoint any one of the judges of that Court as Acting Chief Justice. Appointment of Additional and Acting Judges Under Article 224 (1) if for some temporary reason the functions of a High Court has increased temporarily and the President feels the necessity to increase the number of Judges temporarily for these functions, then he can appoint those who fulfill the requisite qualifications as Additional Judges for a period not exceeding two years. In the like manner, under article 224 (2) during the absence of some judge, the President can appoint acting judge. Transfer of Judges in Other State According to article 222, the President in consultation with the Chief Justice of a India, can transfer a judge from one High Court to any other High Court. Subordinate Courts Criminal Courts: The highest court in the district is that of District and Sessions judge. It is empowered to hear both the civil as well as criminal cases. It should be noted that the district and the session court is one and the same court and the same person acts in both the civil and criminal capacities. When he deals with civil cases, he is known as the District Judge and when he hears criminal cases, he is called the Sessions Judge.

He is appointed by the Governor of the state in consultation with the Chief Justice of the High Court. The appointments to this post are made in two ways. In the first place, those persons are appointed to this post who are not in government service; the persons who have worked continuously as lawyers or advocates for seven years, are appointed. Secondly, government servants in judicial service commission. These government employees include Munsiffs who by getting promotions are appointed. The District and Sessions Court is the highest court of the district. The District and Sessions Judges hears appeals from subordinate courts under it. He can hear appeals regarding grave crimes like dacoities and murder and is empowered to sentence the culprits to death, but such punishment must be confirmed by the High Court. The lowest criminal court in the district, is that of Third class Magistrates Court. This magistrate hears petty cases like those of beating and quarrels etc. He is empowered to sentence one months imprisonment and maximum fine of some petty amount. Civil Court: The Highest Civil Court in the district is that of a District Judge. The same person hears the civil and the criminal cases. When he hears criminal cases he is called the Session Judge; but when he deals with civil cases, he is called the District Judge. There are Courts of many sub-judges under him. They possess original jurisdiction and also can hear appeals against the Munsiffs Court. High Courts by state / union territory State & UTs Andaman and Nicobar Island Arunachal Pradesh Andhra Pradesh Assam Chandigarh Dadra and Nagar Haveli Daman and Diu Court Calcutta High Court Guwahati High Court Andhra Pradesh High Court Guwahati High Court City Kolkata Guwahati Hyderabad Guwahati

Punjab and Haryana High Court Chandigarh Bombay High Court Bombay High Court Mumbai Mumbai New Delhi Mumbai Ahmedabad

National capital territory of Delhi Delhi High Court Goa Gujarat Bombay High Court Gujarat High Court

Himachal Pradesh Jammu and Kashmir Jharkhand Karnataka Kerala Lakshadweep Madhya Pradesh Maharashtra Manipur Meghalaya Mizoram Nagaland Orissa Pondicherry Punjab Rajasthan Sikkim Tamil Nadu Tripura Uttrakhand Uttar Pradesh

Himachal Pradesh High Court

Shimla

Jammu and Kashmir High Court Srinagar/Jammu Jharkhand High Court Karnataka High Court Kerala High Court Kerala High Court Madhya Pradesh High Court Bombay High Court Guwahati High Court Guwahati High Court Guwahati High Court Guwahati High Court Orissa High Court Madras High Court Ranchi Bangalore Ernakulam Ernakulam Jabalpur Mumbai Guwahati Guwahati Guwahati Guwahati Cuttack Chennai

Punjab and Haryana High Court Chandigarh Rajasthan High Court Sikkim High Court Madras High Court Guwahati High Court Uttarakhand High Court Allahabad High Court Jodhpur Gangtok Chennai Guwahati Nainital Allahabad

West Bengal Bihar Benches of the High Courts

Calcutta High Court Patna High Court

Kolkata Patna

Lucknow for Allahabad High Court Nagpur, Panaji, Aurangabad for Bombay High Court Port Blair (circuit bench) Kohima, Aizwal & Imphal. Circuit Bench at Agartala & Shillong for Guwahati High Court. Circuit Benches at Hubli-Dhawrad & Gulbarga for Karnataka High Court Gwalior, Indore for MP High Court Madurai for Tamil Nadu High Court Jaipur for Rajasthan High Court

Attorney General of India


Ques. 1 : What are the rule & functions of Attorney General of India? Ans. The Attorney General for India is appointed by the President of India under Article 76 of the Constitution and holds office during the pleasure of the President. He must be a person qualified to be appointed as a Judge of the Supreme Court. It is the duty of the Attorney General for India to give advice to the Government of India upon such legal matters and to perform such other duties of legal character as may be referred or assigned to him by the President. In the performance of his duties, he has the right of audience in all Courts in India as well as the right to take part in the proceedings of Parliament without the right to vote. In discharge of his functions, the Attorney General is assisted by a Solicitor General and four Additional Solicitors General. The Solicitor General and Additional Solicitors General are not constitutional posts. The salary of the Attorney General is fixed by the President. The Attorney General of India is the Indian governments chief legal advisor. He/She also represents the Government of India in any reference made by the President to the Supreme Court under Article 143 of the Constitution. The Attorney General is to be consulted only in legal matters of real importance and only after the Ministry of Law has been consulted. All references to the Attorney General are made by the Ministry of Law. The Attorney General can practice as a lawyer but cannot appear against the Government. He/She cannot defend an accused in the criminal proceedings and accept the directorship of a company without the permission of the Government. Attorney General of India does not have any executive authority.

Potrebbero piacerti anche